Download as pdf or txt
Download as pdf or txt
You are on page 1of 31

VISIONIAS

www.visionias.in
ANSWERS & EXPLANATION
GENERAL STUIDES (P) TEST – 2967 (2020)

Q 1.D
• Akbar's reign(1556-1605) holds a certain prominence in history; he was the ruler who actually fortified
the foundations of the Mughal Empire.
• Persian prose and poetry were at climax under Akbar's reign.
• Shaikh Abu al-Fazal ibn Mubarak also known as Abu'l-Fazl, was the vizier of the Mughal emperor
Akbar, and author of the Akbarnama, the official history of Akbar's reign in three volumes, (the third
volume is known as the Ain-i-Akbari) and a Persian translation of the Bible. He was also one of the Nine
Jewels (Navaratnas) of Akbar's royal court and the brother of Faizi, the poet laureate of Emperor Akbar.
• Faizi (brother of Abu’l Fazl) was the leading poet of that age. Faizi also worked for the Akbar's
translation department. The translation of the Mahabharata was carried out under his supervision.
• Utbi and Naziri were the two other leading Persian poets during Akbar's reign. They migrated from
Iran to India and made the Mughal court one of the cultural centres of the Islamic world. Naziri became
the first Persian-born poet to join the court of the great Mughal statesman and literary patron ʿAbd-al-
Raḥim Ḵān-e Ḵānān. Besides, Hindus also contributed to the growth of Persian literature.
• Tulsidas (1532-1623) was one of the most influential Hindi poets of the medieval period who had
written Ramcharitmanas. He used a dialect of Hindi, spoken in the eastern parts of Uttar Pradesh
(around Banaras).
• Eknath and Tukaram (1608-50)developed and made the Marathi language popular. Tukaram was a
contemporary of Jahangir (1605-1627). The great Maratha chief Shivaji was an admirer of
Tukaram.

Q 2.A
• The theme of Mara Vijaya has been painted in the caves of Ajanta in cave No. 26.
• It is sculpted near the colossal Buddha image of Mahaparinibbana.
• The panel shows the image of the Buddha in the centre surrounded by Mara’s army along with his
daughter. The event is part of the enlightenment. It is a personification of the commotion of mind which
the Buddha went through at the time of enlightenment. Mara represents desire.
• According to the narrative, there is a dialogue between the Buddha and Mara, and the Buddha is shown
with his right hand indicating towards earth as a witness to his generosity. The composition is very
complex with highly voluminous images. The figure on the right shows Mara coming with his army
consisting of various kinds of people including some with grotesque animal faces. On the left lower end,
the image of Mara is shown contemplating how to disturb Siddhartha, the name of the Buddha before
enlightenment. The army of Mara is shown marching towards the Buddha in the first half of the panel
whereas the lower half of the panel shows the departing army of Mara giving him adorations. The
centrally placed Buddha is in padmasana and a tree at the back is shown by dense leaves.
• About Ajanta caves:
o Ajanta caves are a series of Buddhist caves located in Aurangabad district, Maharashtra. It
encompasses both Theravada and Mahayana Buddhist traditions.
o They are constructed in 2 phases:
▪ Phase I- Satavahana Phase: Since Buddha was against the idea of sculpting and painting images
of him. Hence, this phase depicts the Lord via symbols;
▪ Phase 2 - Vakataka Phase – Major difference from Phase I is the depiction of Buddha as a result
of the spread of Mahayana Buddhism. Here Buddha is represented through images and idols.
Q 3.C
 During the rule of Mughal Emperor Aurangzeb, there was resentment among Hindus for revival of Jaziya
and general destruction of temples under imperial orders. The revolt triggered when a Mughal soldier
killed a Satnami. The Satnamis killed the soldier in revenge and in turn Mughal soldiers were sent to teach
them a lesson. In 1672, at Narnaul, an armed conflict occurred between the peasants and the Mughals.
Some 5,000 Satnamis stood up in arms and routed the Mughal troops in the town, drove away the Mughal
administrators and set up their own administration under their leader Birbhan. Hence Statement 1 is
correct.
 The absence from North India of the major proportion of royal troops helped the revolting people to
establish a temporary gain and the movement of rebels towards Delhi became a matter of serious concern
for the emperor himself.
 The Satnamis were mostly peasants, artisans, and lower caste people, such as Goldsmiths, Carpenters,
Sweepers, Tanners, and other ignoble beings. Hence Statement 2 is correct.
 They did not observe distinction of caste and rank or between Hindus and Muslims and followed a
strict code of conduct.
 It is interesting to note that the local Zamindars, many of whom were rajputs, sided with Mughals in this
conflict.

Q 4.C
 Statement 1 is not correct: Kapila was the founder of the Samkhya School. Based on the Upanishads,
two schools of philosophy developed in India:
o The realistic (e.g. Samkhya)
o The idealistic (e.g.Vedanta).
 The Samkhya philosophy combines the basic doctrines of Samkhya and Yoga. However, the Samkhya
represents the theory and Yoga represents the application or the practical aspects.
 Statement 2 is correct: Samkhya system believes that the soul can attain liberation only through real
knowledge. Real knowledge can be acquired through observation, inference, and words.
 Statement 3 is correct: Kapila's primary principle is that the world is material. Matter (Prakriti) is
the basis of everything that is, it is omnipresent, eternal, and one. The motion of Prakriti is just as eternal
as Prakriti itself. Primordially, Prakriti has no outside cause, for matter (Prakriti) has neither beginning nor
end. Kapila wrote that the world was not created, and therefore there was no creator; the world itself was
the cause of the world; the world developed gradually. Thus he rules out the existence of God. Samkhya
argues that if God exists and if God is eternal and unchanging as is widely claimed, then he can not be the
cause of the world. A cause has to be active and changing.
 Charvaka propounded another materialistic philosophy known as Lokayata.

Q 5.C
 The edicts of Ashoka are the first physical evidence of Buddhism and how it was spread during his region.
 These edicts were deciphered by the British archaeologist and historian James Prinsep.
• The edicts are divided into Pillar Edicts (total 7), Major Rock Edicts(total 14) and 2 separate ones found
in Odisha(Kalinga), Minor Rock Edicts, the queen’s Edict, Barabar Caves inscriptions and the Kandahar
bilingual inscription.
 Sanchi hosts a minor rock edict of Ashokan Inscription, other locations have major rock edicts.
 Girnar - It is situated one mile to the east of Junagarh in Kathiawar. That it was a site of immense
importance is amply proved by the number of major inscriptions to be found there, including apart from
those of Asoka, those of Rudradaman and Skandagupta. It is mentioned as Girinagar in the Brihat
Samhita. By tradition, the mountain is regarded as sacred both to Brahmans and Jainas.
 Sopara - It is situated in the Thane district of Bombay and is the site of an ancient sea-port and town,
which no doubt was of importance during the reign of Asoka. It has been identified with the Soupara of
Ptolemy, described as a commercial centre.
 Kandahar - the site of the inscription is Shar-i-Quan, the old city of Kandahar in southern Afghanistan. It
grew to importance with the establishment of trade between the Hellenic world and north-west India after
the campaigns of Alexander had established contact.
 Option (c) is the correct answer.
Q 6.A
 The origin of Kathak is traditionally attributed to the travelling bards of ancient northern India known
as Kathakars. Kathak evolved during the Bhakti movement, particularly by incorporating the
childhood and stories of the Hindu god Krishna, as well as independently in the courts of north Indian
kingdoms.
 Statement 1 is correct: The emergence of Raslila, mainly in the Braj region (Mathura in Western U.P.)
was important in its development. It combined in itself music, dance and the narrative. Dance in
Raslila, however, was mainly an extension of the basic mime and gestures of the Kathakars or story-
tellers which blended easily with the existing traditional dance.
 Statement 2 is correct: Kathak has emerged as a distinct dance form. Being the only classical dance of
India having links with Muslim culture, it represents a unique synthesis of Hindu and Muslim genius in
art. Further, Kathak is the only form of classical dance wedded to Hindustani or the North Indian
music. Both of them have had a parallel growth, each feeding and sustaining the other.
 Statement 3 is not correct: The Mughal era courts and nobles accepted Kathak as a form of aristocratic
entertainment, which low-income families were willing to provide. However, the dance became more
abstract and erotic, less as a means of communication of spiritual or religious ideas.

Q 7.A
 The right of ownership of Zamindars regarding the land depended mainly on succession. Hence
statement 1 is correct.
 The people who settle a new village or who brought wastelands under cultivation, belong to the respective
villages. These villagers became the owners of these lands.
• The considerable section of the zamindars had the hereditary right of collecting land revenue from their
respective villages. This was called his ‘talluqa’ or his ‘zamindari.’
 For collecting the land revenue, the zamindars received a share of the land revenue which could go up to
25 percent.
 Zamindars held enormous tracts of land and control over their peasants, from whom they reserved
the right to collect tax on behalf of imperial courts or for military purposes.
• The zamindars, not necessarily “owner” of all the lands over which he collected the land revenue.
Hence statement 2 is not correct.
 The peasants who actually cultivated the land could not be dispossessed as long as they paid the land
revenue. Thus the zamindars and the peasants, both had their own hereditary rights in land.

Q 8.A
 The Vijayanagara Empire was based in the Deccan Plateau region in South India. The empire is named
after its capital city of Vijayanagara, whose ruins surround present day Hampi, now a World Heritage Site
in Karnataka, India. It was established in 1336 by Harihara I and his brother Bukka Raya I of Sangama
Dynasty.
• The first dynasty, the Sangama, lasted until about 1485, when at a time of pressure from the Bahmanī
sultan and the raja of Orissa, Narasimha of the Saluva dynasty usurped power.
• By 1503 the Saluva dynasty had been supplanted by the Tuluva dynasty. The outstanding Tuluva
king was Krishna Deva Raya. During his reign (1509–29) the land between the Tungabhadra and Krishna
rivers (the Raichur doab) was acquired (1512), the Orissa Hindus were subdued by the capture of
Udayagiri (1514) and other towns, and severe defeats were inflicted on the Bijapur sultan (1520).
• Krishna Deva’s successors, however, allowed their enemies to unite against them. In 1565 Rama Raya,
the chief minister of Vijayanagar, led the empire into the fatal battle at Talikota, in which its army was
routed by the combined forces of the Muslim states of Bijapur, Ahmadnagar, and Golconda and the city of
Vijayanagar was destroyed.
 Tirumala, brother of Rama Raya, then seized control of the empire and founded the Aravidu
dynasty, which established a new capital at Penukonda and kept the empire intact for a time.
 Hence, option (a) is the correct answer.

Q 9.A
 Indian classical music has two major traditions - the North Indian classical music tradition is called
Hindustani, while the South Indian expression is called Carnatic. In Haripala's "Sangeeta
Sudhakara", written in the 14th century A.D., the terms Carnatic and Hindustani are found for the first
time. The Indian music of the Northern part of India assimilated some features of the music of the Persian
and Arabic musicians who adorned the courts of the Mughal rulers of Delhi, the music of the South
continued to develop along its own original lines. Hence statement 1 is correct.
 In Hindustani music, a gharana is a system of social organization in the Indian subcontinent, linking
musicians or dancers by lineage or apprenticeship, and by adherence to a particular musical style.
Hence statement 2 is correct.
 The major vocal forms associated with Hindustani classical music are the Khayal, Ghazal, Dhrupad,
Dhammar, tarana and Thumri. Dhrupad style of singing is traditionally performed by men with a
tanpura and pakhawaj. The lyrics sung in Dhrupad are in a medieval form of Hindi and typically heroic in
theme, or in praise of a particular deity. A more adorned form is called Dhamar. Khayal consists of
about 4-8 lines of lyrics set to a tune. The performer uses these few lines as the base for
improvisation. Taranas are songs that are used to convey a feel of joy and are usually performed towards
the end of a concert.
 Although vocal music plays an important role, instrumental music is more important in Hindustani music.
There are some purely instrumental forms, such as the theme with variations known as gat. Hence
statement 3 is not correct.

Q 10.B
 About Gommateshwara Statue
o It is a Jain shrine at Shravanbelagola, Karnataka dedicated to Lord Bahubali. However, he could not
become a Tirthankara. Last Jain Tirthankara is Lord Mahavira. Hence statement 1 is not correct.
o Standing at fifty-seven feet high, it is is the world’s tallest monolithic free-standing structure made up
of the granite. Hence statement 2 is correct.
o It was commissioned by Camundaraya, the General-in-Chief and Prime Minister of the Ganga Kings
of Mysore. Hence statement 3 is not correct.
o About Gangas of Mysore:
▪ They are also called Western Gangas to distinguish them from the Eastern Gangas who in later
centuries ruled over Kalinga.
▪ They ruled in ancient Karnataka from about 350 to 1000 CE.
▪ Camundaraya is said to be inspired by Jain Acarya, Nemichandra Siddhānta Chakravartin to build
the statue of Bahubali.

Q 11.C
 The most important economic development of the Post Mauryan period was the thriving trade between
India and the eastern Roman empire. In the beginning, a good deal of this trade was carried on by land,
but the movement of the Shakas, Parthians and Kushans beginning from the first century B.C. disrupted
trade by land route. Since the first century A.D. trade was carried on mainly by the sea.
 The Shaka and Kushan kingdoms used two routes from the north-western frontier to the western sea coast.
Both these routes converged at Taxila, and were connected to the Silk Route passing through Central
Asia. The first route directly ran from the north to the south connecting Taxila with the lower Indus basin
from where it passed on to Broach.
 The second route called Uttarapath was in more frequent use. From Taxila it passed through modern
Punjab up to the western coast of Yamuna. Following the course of the Yamuna, it went southward
to Mathura. From Mathura it passed on to Ujjain in Malwa and again from Ujjain to Broach on the
western coast. Ujjain was the meeting point of another route which started from Kaushambi near
Allahabad. Mahishmati lies on the Dakshinapath. Hence option (c) is the correct answer.

Q 12.A
 Statement 1 is correct: In sixth century BCE plough Agriculture was spread in fertile alluvial river
valleys like the Ganga and the Kaveri. The iron-tipped ploughshare was used to turn the alluvial soil in
areas which had high rainfall. In some parts of the Ganga valley, production of paddy was dramatically
increased by the introduction of transplantation.
• Statement 2 is not correct: Use of iron ploughshare led to a growth in agricultural productivity, its use
was restricted to certain parts of the subcontinent – cultivators in areas which were semi-arid, such as
parts of Punjab and Rajasthan did not adopt it till the twentieth century, and those living in hilly tracts in
the north-eastern and central parts of the subcontinent practised hoe agriculture.

Q 13.A
• Pandavani is a folk singing style involving narration of tales from the ancient Indian epic Mahabharata.
The singing also involves musical accompaniment. Bhima, the second of the Pandava is the hero of the
story in this style. In Pandavani, tales from Mahabharata are sung as a ballad and one or two episodes
are chosen for the night’s performance. The main singer continuously sits throughout the performance and
with powerful singing and symbolic gestures he assumes all the characters of the episode one after
another. This form of folk theatre is popular in the central Indian state of Chhattisgarh. Hence, pair 1 is
correctly matched.
 Mando is a musical form that evolved during the 19th and 20th century among Goan Catholics of Goa,
India. It represents the meeting point of Indian and western musical traditions. It is a slow verse and
refrain composition dealing with love, tragedy and both social injustice and political resistance
during Portuguese presence in Goa. Hence pair 2 is correctly matched.
 Laman is a part of folk music of Himachal Pradesh. In Laman a group of girls sing a stanza and a
group of boys give reply in the song. This continues for hours. Interesting is that the girls singing on one
of the peaks of the hill seldom see the faces of the boys singing on another peak. In between is the hill
which echoes their love song. Most of these songs are sung especially in Kullu Valley. Hence pair 3 is
not correctly matched.

Q 14.C
 Statement 1 is not correct: Turkish troops did not have superior weapons as compared to
Indians. Turkish bows could shoot arrows to a longer distance, but the Indian bows were supposed to be
more accurate and more deadly, the arrowheads being generally dipped in poison. In hand to hand
combat, the Indian swords were considered to be the best in the world.
 Statement 2 is not correct: The Indians had the advantage of elephants. However, Turks had horses
which were swifter and more sturdy than the horses imported into India.
 Statement 3 is correct: The Turks were more socially and organisationally superior. The growth of
feudalism, that is rise of local landed elements and chiefs weakened the administrative structure and
military organisation of the Indian states. The rulers had to depend more on the various chiefs who rarely
acted in coordination and quickly dispersed to their areas after the battle. On the other hand, the tribal
structure of the Turks and the growth of iqta and khalisa system enabled the Turks to maintain
large standing army which could be kept in the field for a long time.
 The Indians were not effective to move as an organised body of horseman which covered long distances
and fight. Rajputs put up prolonged resistance to the incursions of Arab and Turks but they never tried to
be offensive and try to push the Arabs or Turks from the strategic lands like Afghanistan, Punjab that they
had occupied. That is to say, Rajputs lacked a strategic vision.

Q 15.B
 Statement 1 is not correct: Satavahanas ruled over parts of western and central India ( c. second
century BCE -second century CE). Both Satvahanas and the Shakas, a people of Central Asian origin
who established kingdoms in the north-western and western parts of the subcontinent, derived revenues
from long-distance trade. Their social origins were often obscure, but in the case of the Satavahanas, once
they acquired power they attempted to claim social status in a variety of ways.
 Statement 2 is correct: Satavahana rulers were identified through metronymics (names derived from that
of the mother). Although this may suggest that mothers were important, in the case of the
Satavahanas the succession to the throne was generally patrilineal e.g. raja Gotami-puta Siri-
Satakani, the names had uniform title raja, with the term puta, a Prakrit word meaning 'son'. The
term Gotami-puta means 'son of Gotami'.
• Statement 3 is not correct: Many Kushana rulers and not Satavahana rulers adopted the title devaputra,
or “son of god”, possibly inspired by Chinese rulers who called themselves sons of heaven. This was a
means of claiming a high status. Colossal statues of Kushana rulers have been found installed in a shrine
at Mat near Mathura (Uttar Pradesh). Similar statues have been found in a shrine in Afghanistan as well.
Some historians feel this indicates that the Kushanas considered themselves godlike.

Q 16.B
 About Brihadishvara Temple
o Also called Rajarajesvaram, it is a Hindu temple dedicated to Shiva located in Thanjavur, Tamil
Nadu, India
o It was completed around 1009 by Rajaraja Chola.
o It is in this temple that one notices for the first time two large gopuras (gateway towers) with an
elaborate sculptural program which was conceived along with the temple.
o The main deity of the temple is Shiva, who is shown as a huge lingam set in a two storeyed
sanctum.
 Rajendra Chola, who was the son and successor of Rajaraja Chola built the Gangaikondacholapuram
which is located at Thanjavur (Tanjore) in Tamil Nadu.
Q 17.B
• Buddha’s teachings and discourses were included in the Sutta Pitaka. In it there is mention of the
advice given by the Buddha to a wealthy householder named Sigala on how a master look after his
servants and employees i.e. by assigning them to work according to their strength, by supplying them
with food and wages, by tending them in sickness; by sharing delicacies with them and by granting leave
at times. It also contains:
o The five ways should the clansmen look after the needs of samanas (those who have renounced the
world) and Brahmanas
o There are similar instructions to Sigala about how to behave with his parents, teacher and wife
 The Sutta Pitaka is grouped into five nikayas or collections:
o Digha Nikaya: The "Long" Discourses
o Majjhima Nikaya: The "Middle-length" Discourses
o Samyutta Nikaya: The "Grouped" Discourses
o Anguttara Nikaya: The "Further-factored" Discourses consists of several thousand short suttas,
grouped together into eleven nipatas according to the number of items of Dhamma covered in each
sutta. It also consists of a list of 16 Mahajanapadas
o Khuddaka Nikaya: The "Division of Short Books"
 Detailed information about other Tripitikas:
o The Vinaya Pitaka: It includes rules and regulations for those who joined the sangha or monastic
order. It includes not only the rules governing the life of every Theravada bhikkhu (monk) and
bhikkhuni (nun), but also a host of procedures that support harmonious relations, both among the
monastics themselves, and between the monastics and their lay supporters. It consist of 3 parts:
▪ Suttavibhanga — the basic rules of conduct (Patimokkha) for bhikkhus and bhikkhunis, along
with the "origin story" for each one.
▪ Khandhaka:
❖ Mahavagga — in addition to rules of conduct and etiquette for the Sangha, this section
contains several important sutta-like texts, including an account of the period immediately
following the Buddha's Awakening, his first sermons to the group of five monks, and stories
of how some of his great disciples joined the Sangha and themselves attained Awakening.
❖ Cullavagga — an elaboration of the bhikkhus' etiquette and duties, as well as the rules and
procedures for addressing offences that may be committed within the Sangha.
❖ Parivara — A recapitulation of the previous sections, with summaries of the rules classified
and re-classified in various ways for instructional purposes.
o Abhidhamma Pitaka: Abhidhamma Pitaka dealt with philosophical matters. Each Pitaka comprised
a number of individual texts.The Abhidhamma Pitaka is divided into seven books, although it is the
first (Dhammasangani) and last (Patthana) that together lay out the essence of Abhidhamma
philosophy. The seven books are:
▪ Dhammasangani ("Enumeration of Phenomena").
▪ Vibhanga ("The Book of Treatises")
▪ Dhatukatha ("Discussion with Reference to the Elements")
▪ Puggalapaññatti ("Description of Individuals")
▪ Kathavatthu ("Points of Controversy")
▪ Yamaka ("The Book of Pairs").
▪ Patthana ("The Book of Relations").
o Madhyamika: Mādhyamika is an important school in the Mahāyāna (“Great Vehicle”) Buddhist
tradition. Its name derives from its having sought a middle position between the realism of the
Sarvāstivāda (“Doctrine That All Is Real”) school and the idealism of the Yogācāra (“Mind Only”)
school. The most renowned Mādhyamika thinker was Nāgārjuna (2nd century AD), who developed
the doctrine that all is void (śūnyavāda).

Q 18.A
 Kuchipudi is a dance-drama performance (Andhra Pradesh), with its roots in the ancient Hindu
Sanskrit text of Natya Shastra. It was developed as a religious art linked to traveling bards, temples and
spiritual beliefs, like all major classical dances of India.
 The Kuchipudi performance usually begins with an invocation. Then, each costumed actor is introduced,
their role stated, and they then perform a short preliminary dance set to music (dharavu). Next, the
performance presents pure dance (nritta). This is followed with by the expressive part of the performance
(nritya), where rhythmic hand gestures help convey the story. Vocal and instrumental Carnatic music
in the Telugu language accompanies the performance. The typical musical instruments in Kuchipudi
are mridangam, cymbals, veena, flute and the tambura.
 To show the dexterity of the dancers in footwork and their control and balance over their
bodies, techniques like dancing on the rim of a brass plate and with a pitcher full of water on the
head was introduced. Acrobatic dancing became part of the repertoire. At present Kuchipudi fully
crystallized as a separate classical solo dance style. Thus there are now two forms of Kuchipudi - the
traditional musical dance-drama and the solo dance.
 Hence all the three movements/traditions are associated with Kuchipudi dance whereas colourful masks
are not used in it. Hence option (a) is the correct answer.

Q 19.A
 Niccolao Manucci was an Italian doctor, writer and traveller. He wrote a memoir about the Indian
subcontinent during the Mughal era. His records have been a source of history about Shah
Jahan, Aurangzeb, Shivaji Maharaj, Dara Shikoh, Shah Alam, Raja Jai Singh and Kirat Singh.
 He is famous for his work "Storia do Mogor", an account of Mughal history and life. Manucci had first-
hand knowledge of the Mughal court, and the book is considered to be the most detailed account of the
Mughal court. It is an important account of the time of the later reign of Shah Jahan and of the reign
of Aurangzeb.
 He spent almost all his adult life in India and breathed his last in Chennai. He arrived in Surat as a
17-year-old in 1656 and never went back.
 Hence, option (a) is the correct answer.
 Duarte Barbosa
o He was a Portuguese trading agent at Cannanore and Cochin in between 1503 and 1517 and had left
behind an interesting account on trade and political events of the southeast including Bengal.
 Francois Bernier
o François Bernier was a French physician and traveller. He was briefly personal physician to Mughal
prince Dara Shikoh , the eldest son of the Mughal emperor Shah Jahan, and after Dara Shikoh's
demise, was attached to the court of the Mughal emperor Aurangzeb.
o He wrote Travels in the Mughal Empire, which is mainly about the reigns of Dara Shikoh and
Aurangzeb. It is based on his own extensive journeys and observations, and on information from
eminent Mughal courtiers who had witnessed the events at first hand.
 Marco Polo
o Marco Polo was an Italian merchant, explorer, and writer, born in the Republic of Venice. He
travelled to India while returning home to Venice from China. He arrived on the Coromandel Coast of
India in a typical merchant ship with over sixty cabins and up to 300 crewmen. He marveled at the
Indian culture and wealth.

Q 20.D
 Akbar tried to emphasize the concept of sulh-kul or peace and harmony among religions in different
ways.
 He set up a big translation department for translating works in Sanskrit, Arabic, Greek, etc., into
Persian. Thus, the Singhasan Battisi, the Atharva Veda and the Bible were taken up first for translation.
 These were followed by the Mahabharata, the Gita and the Ramayana.
 Many others, including the Panchatantra and works of geography, were also translated.
 The Quran was also translated perhaps for the first time.

Q 21.A
 Trade was facilitated by the use of money. The coin or metal money bearing the stamp of an authority was
invented in the seventh century BC in Lydia in Asia Minor. How it was first introduced in India is not
clear. The terms nishka and satamana in the Vedic texts are taken to be names of coins, but they seem to
have been prestige objects made of metal.
 It appears that in Vedic times, exchange was conducted through barter, and the mutual gift system
served as a mode of exchange in pre-Buddhist times. Sometimes cattle served the purpose of
currency. Coins made of metal appear first in the age of Gautama Buddha.The earliest were made
largely of silver, though a few copper coins also existed. They are called punch-marked because pieces
of silver and copper were punched with certain marks, such as a hill, tree, fish, bull, elephant, and
crescent. Hence statement 1 is correct.
 The large, long-service army had to be fed by the state exchequer. The Nandas possessed enormous
wealth which must have enabled them to maintain the army with a well-established fiscal system.
Warriors and priests, that is, the kshatriyas and the brahmanas, were exempted from payment of
taxes, and the burden fell on the peasants who were mainly vaishyas or grihapatis. Bali, a voluntary
payment made by the tribesmen to their chiefs in Vedic times, became a compulsory payment to be
made by the peasants in the age of the Buddha, and officers called balisadhakas were appointed to
collect it. Hence statement 2 is not correct.
 It appears that one-sixth of the produce was collected as tax by the king from the peasant. Taxes were
assessed and collected by the royal agents with the help of village headmen.

Q 22.A
• Pair 1 is not correctly matched: When the land left uncultivated for one year, it was called ‘parati’
(fallow). Cess on Parati land was at the full (polaj) rate when it was cultivated.
• Pair 2 is not correctly matched: The land which had been fallow for two to three years was called
‘chachar’, and if longer than that, it was known as ‘banjar’.
• Pair 3 is correctly matched: The land which remained under cultivation almost every year was
called ‘polaj’.
 The land was also classified as good, middling, and bad. Though one-third of the average produce was
the state demand, it varied according to the productivity of the land, the method of assessment, etc.
 Akbar was deeply interested in the development and extension of cultivation; therefore, he offered
taccavi (loans) to the peasants for seeds, equipment, animals, etc. Akbar made policy to recover the loans
in easy installments.

Q 23.C
 The Deccani states had a number of cultural contributions to their credit.
 Ali Adil Shah was very fond of organizing discussions with Hindu and Muslim saints.
 Adil Shah invited Catholic missionaries to his court, much before Akbar had done so. He had an
excellent library to which he appointed the well-known Sanskrit scholar, Waman pandit. Patronage of
Sanskrit and Marathi was continued by his successors.
 Ibrahim Adil Shah II (1580-1627), the successor of Adil Shah, ascended the throne (of Bijapur) at the
age of nine. He was very attentive of the poor, and had the "title of Abla Baba", or Friend of the Poor.
 Adil Shah II was very fond of music; he composed a book namely Kitab-i-Nauras (Book of
Nine Rasas).
 In this book, he set various musical modes or togas. In his songs, he freely prayed the goddess of music
and learning, Saraswati. Due to his broad approach, he came to be called as Jagat Guru.
 Adil Shah II, further, built a new capital, Nauraspur; where he invited a large number of musicians (to
settle).
 He offered patronage to all, including Hindu saints and temples. This included grants to Pandharpur,
the center of the worship of Vithoba, which became the center of the Bhakti movement in Maharashtra.

Q 24.B
• Silappadikaram, (Tamil: “The Jeweled Anklet”) also spelled Silappatikaram, the earliest epic poem in
Tamil, written in the 5th–6th century AD by Prince Ilanko Adikal (Ilango Adigal). The
Silappadikaram tells about the young merchant Kovalan’s marriage to the virtuous Kannaki
(Kannagi), his love for the courtesan Madhavi, and his consequent ruin and exile in Madurai, where
he is unjustly executed after trying to sell his wife’s anklet to a wicked goldsmith who had stolen the
queen’s anklet and charged Kovalan with the theft.
• The Silappathikaram is a fine synthesis of mood poetry in an ancient Tamil Śaṅgam tradition and the
rhetoric of Sanskrit poetry, including the dialogues of Kalittokai (poems of unrequited or mismatched
love), chorus folk song, descriptions of city and village, lovingly technical accounts of dance and music,
and strikingly dramatic scenes of love and tragic death.
• Unlike the Silappathikaram, its incomplete sequel, Manimekalai, the story of Kovalan’s and Madhavi’s
daughter, reflects a Buddhist perspective.

Q 25.C
 Kuru basically belonged to the Puru-Bharata family. These were the people who originated
from Kurukshetra (Northern India ). They are believed to have shifted to the republic form of
government in the fifth of the sixth century BCE.
• Koshala embraced the area occupied by eastern UP and had its capital at Shravasti, which is
coterminous with Sahet–Mahet on the borders of Gonda and Bahraich districts of UP.Koshala had
an important city called Ayodhya which is associated with the story in the Ramayana.
 Chedi people existed on the southern part of the Yamuna River. The capital of Chedis was Suktimati
and the Rigveda gives details of this Mahajanapada. It was ruled by Sisupala and it was this city where
the Pandavas (from Mahabharata) chose to spend the thirteenth year of their exile.
 Assaka or Ashmaka, was a region of ancient India around and between the river Godavari. It was one
of the shodasa mahajanapadas in the 6th century BCE, mentioned in the Buddhist text Anguttara Nikaya.
The capital is variously called Potali, or Podana, which now lies in the Nandura Tehsil of Maharastra.

Q 26.A
 According to the Abhinaya Darpanam, the Sangitaratnakara and other medieval treatises, dance is divided
into three distinct aspects, that is, natya, nritya and nritta.
 Natya corresponds to drama. it is the combined manifestation of bhava, rasa and abhinaya. Natya
highlights the dramatic element and most dance forms do not give emphasis to this aspect today with the
exception of dance-drama forms like Kathakali. Hence statement 1 is correct.
 Nritya consists of footwork and abhinaya. It relates to Rasa and psychological state. Angika abhinaya
relating to Hasta, eyes, eye brows, lips etc. are very important in Nritya. It can be termed as the
explanatory aspect of dance where hand gestures and facial expressionsconvey the meaning of the lyrics
of the performing song. Bhav of the dancer is of prime importance in this so it can also be considered as
the miming aspect of dance. Hence statement 2 is not correct.
 Nritta corresponds to pure dance steps performed rhythmically. Here the movements of the body do
not convey any mood or meaning and its purpose is just creating beauty by making various patterns, lines
in space and time. Hence statement 3 is not correct.

Q 27.A
• One of Ashoka’s first artistic programs was to erect the pillars that are now scattered throughout what was
the Mauryan empire. The pillars vary from 40 to 50 feet in height. They are cut from two different types
of stone—one for the shaft and another for the capital. The shaft was almost always cut from a single
piece of stone. Laborers cut and dragged the stone from quarries in Mathura and Chunar, located in the
northern part of India within Ashoka’s empire.
 Pillar inscriptions were found at Pillars of Ashoka is located in India at topara kalan , Meerut , Sankissa ,
Allahabad , Sanchi , Sarnath, Vaishali , Rampurva , Lauriya Araraj , Lauriya nandangarh and
Ahraura. However some of the pillar inscriptions are also found outside India namely Nigalisagar
(Nepal) and Rummindei (Nepal). Hence option(a) is not correct.
 Some of the Ashokan rock edicts are located outside India namely : Kandhar (Afghanistan), Mansehra,
Shahbazgari (Pakistan). These are written in Kharosthi .The Aramaic and Greek scripts were used for
inscriptions in Afghanistan. Hence option (d) is correct.
• The name Ashoka occurs in copies of Minor Rock Edict I found at three places in Karnataka and at one in
MP. Thus, altogether, the name Ashoka occurs four times. It is significant that Ashoka’s name does not
occur in any of his inscriptions from north or north-west India. The inscriptions which do not carry his
name mention only devanampiya piyadasi, dear to the gods, and leave out the name Ashoka. The
title devanampiya or ‘dear to gods’ adopted by Ashoka was not unique but also adopted by his
ancestors. However, piyadasi or ‘good looking’ seems to have been his unique title. Hence option (b)
is correct.
 Altogether,Ashokan inscriptions appear at forty-seven places, and the total number of versions is 182
including two edicts which are considered spurious. It is significant that Ashokan inscriptions which
were generally located on ancient highways, They have been found at six places in Afghanistan. Hence
option (c) is correct.

Q 28.B
• The Akbar Nama was written to provide a detailed description of Akbar’s reign in the traditional
diachronic sense of recording politically significant events across time, as well as in the more novel sense
of giving a synchronic picture of all aspects of Akbar’s empire – geographic, social, administrative and
cultural – without reference to chronology.
• Abu’l Fazl wrote in a language that was ornate and which attached importance to diction and rhythm, as
texts were often read aloud. This Indo- Persian style was patronised at court, and there were a large
number of writers who wanted to write like Abu’l Fazl.
 The Akbar Nama is divided into three books of which the first two are chronicles.
 The third book is the Ain-i Akbari.
• The first volume contains the history of mankind from Adam to one celestial cycle of Akbar’s life (30
years).
 The second volume closes in the fortysixth regnal year (1601) of Akbar.
• The very next year Abu’l Fazl fell victim to a conspiracy hatched by Prince Salim, and was murdered by
his accomplice, Bir Singh Bundela.
 In the Ain-i Akbari the Mughal Empire is presented as having a diverse population consisting of
Hindus, Jainas, Buddhists and Muslims and a composite culture.
 Ain-i-Akbari embodies Ains or rules and regulations framed and put into effect for proper
administration by Akbar. It is regarded as an administrative manual and is like a modern gazetteer. The
regulations embodied in the Ain-i-Akbari provide information about Akbar's government, and several
departments, its different ranks etc.
 Abul Fazl also discusses in this book the social condition, literary activities, and study of law and
philosophy not only of Muslims but also of local Hindus, Jains and other communities. There are, in
addition, chapters on distinguished travellers, Muslim saints, and sufis.

Q 29.B
 Within the empire, the king appointed rajukas for the administration of justice in his empire, who
were vested with the authority not only to reward people but also to punish them when
necessary. Hence statement 1 is not correct.
 The Maurya period constitutes a landmark in the system of taxation in ancient India. Kautilya
names many taxes which were collected from peasants, artisans, and traders. This required strong and
efficient machinery for assessment, collection, and storage. The Mauryas attached greater importance
to assessment than to storage and deposit. The samaharta was the highest officer in charge of
assessment and collection, and the sannidhata was the chief custodian of the state treasury and
storehouse. The assessor-cum-collector was far more important than the chief treasurer. The damage
inflicted on the state by the first was thought to be more serious than any inflicted by the second.
 In fact, a piece of elaborate machinery for assessment was first set up during the Maurya period. The
list of taxes mentioned in the Arthashastra is impressive, and, if these were really collected, very little
would have been left to the people to live on. Hence statement 2 is correct.
 The secret service or the spy system may be described as the mainstay of the Mauryan government,
next to the army. The king employed hosts of spies or detectives, masquerading in disguises of all
kinds, who were controlled by an espionage bureau. Cipher writing was used and the services of carrier
pigeons were enlisted. Hence statement 3 is correct.
 The Mauryan administration was conducted by a highly skilled Superintendents or Adhyakshas who
looked after various departments. Kautilya in the second book of his Arthasastra, Adhyakshaprachara,
gives an account of the working of nearly 27 adhyaksas.
 Some of the important officials are mentioned below :
o The Akshapataladhyaksha was the Accountant-General who was in charge of the two offices of
currency and accounts.
o The Sitadhyaksha was the superintendent of the agriculture of crown lands or government
agricultural farms.
o Navadhyaksha was the Superintendent of Ports who controlled traffic and transit by waterways.
o The Panyadhyaksha was the controller of commerce who was in the charge of the control of supply,
purchase and sale of commodities.
o The Sulkadhyaksa was the collector of customs and tolls.
o The Suradhyaksha was the Superintendent of Excise who controlled the manufacture and sale of
liquor.
o Pautavadhyaksha was the superintendent of weights and measures. The Lakshanadhyaksha was the
superintendent of the mint, etc.

Q 30.A
 The Mughals made a distinctive contribution in the field of painting. They introduced many new themes
portraying the court, battle grounds, and the chase scenes. Besides, Mughal painters also introduced many
new colors and new forms.
• Statement 1 is correct: During Jahangir’s period, some of the historians claimed that Jahangir had the
sense to distinguish the work of each artist separately in a picture.
 Jahangir himself was a great painter of his times.
 Statement 2 is correct: During those days, it was a fashion in the Mughal School that in a single painting
- the face, the body, and the feet of a person to be painted by different artists.
• During Jahangir’s period, special progress was made in portrait painting and paintings of animals.
Mansur was the great name in this field.
• During the Akbar’s reign, the two great painters (who came India with Humayun), organized painting in
one of the imperial establishments. Besides, a large number of painters from different parts of the country
were invited; many of them were from the lower castes.
 From the beginning, both Hindus and Muslims painters joined in the work. Daswant and Basawan both
were the famous painters of Akbar's court. Hence statement 3 is not correct.
• Mughal painting was at climax under Jahangir’s period who had a very peculiar sense of paintings.

Q 31.B
 The central factor that ultimately transformed the ancient Indian society into medieval society was the
practice of land grants. Land grants became frequent from the 5th Century A.D. According to this, the
brahmanas were granted villages free from taxes. All the taxes which were collected by the king from the
villages were transferred to the brahmanas. In addition to this, the beneficiaries were given the right to
govern the people living in the donated villages. Villages were granted to the brahmanas forever so that
the power of the king was heavily undermined from the end of the Gupta period. Royal control was
further eroded through the payment of government officials by land grants. Hence statement 1 is not
correct and statement 2 is correct.
 In the Mauryan period, the officers of the state from the highest to the lowest were generally paid in cash.
The practice continued under the Kushanas, who issued a large number of copper and gold coins, and it
lingered under the guptas.
 But from the Sixth century A.D., the position changed. The law-books of the time recommended that
services should be rewarded in the land. Accordingly, from the time of Harshavardhana, public officials
were paid in land revenues. A fourth of the royal revenue was earmarked for the endowment of great
public servants.

Q 32.C
 Sun temple at Modhera
o It was built in Gujarat by Raja Bhimdev I of the Solanki Dynasty in 1026.
o It has a massive rectangular stepped tank called the Surya Kund in front of it.
o The Solankis were a branch of the later Chalukyas.
 Martand Sun Temple
o It was built in Jammu and Kashmir by Karkota Dynasty during the 8th century CE.
Martand is another Sanskrit name for the Hindu Sun-god.
o
 Somnath
o The principal temple is believed to have been built in gold by the moon god Soma, in silver by the sun
god Ravi, in wood by Krishna and in stone by the Solanki Rajputs in the 11th century.
o The present temple, built in 1951, is the seventh reconstruction on the original site.
 Dilwara Jain Temples
o It is one of the finest Jain temple located in Rajasthan.
o There are five shrines that are dedicated to Lord Adinath, Lord Rishabhdeo, Lord Neminath, Lord
Mahavir Swami and Lord Parshvanath respectively.

Q 33.B
 The Trinity of Carnatic music, also known as The Three Jewels of Carnatic music, refer to the
outstanding trio of composer-musicians of Carnatic music in the 18th century, being Tyagaraja,
Muthuswami Dikshitar and Syama Sastri. Compositions of the Trinity of Carnatic music are
recognized as being distinct in style, and original in handling ragas. They were patronised and supported
by the king of Tanjore.
• Tyagaraja (1767 - 1847): Tyagaraja composed thousands of devotional compositions, most in Telugu
and in praise of Lord Rama, many of which remain popular today. Of special mention are five of his
compositions called the Pancharatna Kritis which are often sung in programs in his honour. Tyagaraja saw
the reigns of four kings of the Maratha dynasty — Tulaja II (1763–1787), Amarasimha (1787–1798),
Serfoji II (1798–1832) and Sivaji II (1832–1855), although he served none of them.
 Muthuswami Dikshitar (1775 - 1835): His compositions, of which around 500 are commonly known,
are noted for their elaborate and poetic descriptions of Hindu gods and temples and for capturing the
essence of the raga forms through the vainika (veena) style that emphasises gamakas. They are typically
in a slower speed (chowka kala). He is also known by his signature name of Guruguha which is also his
mudra and can be found in each of his songs. He visited and composed ragas at the temples at Kanchi,
Tiruvannamalai, Chidambaram, Tirupathi and Kalahasthi, Srirangam, before returning to Tiruvarur.
• Shyama Shastri (1762–1827): Shyama Shastri adopted common as well as rare ragas for his
compositions. Though his compositions were in three languages - Tamil, Telugu, and Sanskrit - he
preferred Telugu as the medium of expression. He is the only one among the ‘Trinity’ to have composed
in Tamil. Shyama Shastri signed his compositions - Kritis, Varṇam(s) and Swarajati(s) with his Mudra
(Signature) ‘Shyama Krishna’. He was sponsored and supported by the king of Tanjore.
 Annamayya (1408 - 1503) was a 15th-century Hindu saint and is the earliest known Indian musician to
compose songs called sankirtanas in praise of the god Venkateswara, a form of Vishnu. He is the first
known composer in carnatic music. Other composers likePurandaradasa came after him. The musical
form of the keertana songs that he composed, which are still popular among Carnatic music concert
artists, have strongly influenced the structure of Carnatic music compositions. He saw the reign of
Vijayanagar kingdom under the rule of Saluva Narasimha Raya.

Q 34.D
 The rural administration was in the hands of the village headmen. Initially the headmen functioned as
leaders of the tribal regiments, and were therefore called gramini which means the leader of the grama
or a tribal military unit. As life became sedentary and plough cultivation well-established, tribal
contingents settled down to agriculture. The gramini was therefore transformed into a village headman in
pre-Maurya times. Hence statement 1 is correct.
 The village headmen were known by a variety of titles such as gramabhojaka, gramini or gramika. The
title gramini prevails in Sri Lanka to this day. Eighty-six thousand gramikas are said to have been
summoned by Bimbisara. The number may be conventional, but it shows that the village headmen
enjoyed considerable importance and had direct links with the kings.
 The village headmen assessed and collected taxes from the villagers and also maintained law and
order in their locality. Sometimes oppressive headmen were taken to task by the villagers.Hence
statement 2 and 3 are correct.

Q 35.D
 The country Bharatavarsha was eventually named after the tribe Bharata, which is first mentioned in
the Rig Veda. The Bharata ruling clan was opposed by a host of ten chiefs, five of whom were heads of
Aryan tribes and the remaining five of the non-Aryan people. The battle that the Bharatas fought with the
host of ten chiefs is known as the Battle of Ten Kings.
 It was fought on the river Parushni, coterminous with the river Ravi, and it gave victory to Sudas and
established the supremacy of the Bharatas. Of the defeated tribes, the most important was that of
the Purus.
 Hence statements 1 and 2 are not correct.

Q 36.A
 Ayatasra, Kuta and vritta are some of the subdivisions of Dravida temples based upon the shape. These
are basically of five different shapes:
o Square, usually called kuta, and also caturasra;
o Rectangular or shala or ayatasra
o Elliptical, called Gaja-prishta or elephant backed, or also called vrittayata, deriving from
wagon vaulted shapes of apsidal chaityas with a horse-shoe shaped entrance facade usually called a
nasi
o Circular or vritta
o Octagonal or ashtasra.
 Generally, the plan of the temple and the shape of the vimana were conditioned by the iconographic
nature of the deity.
 So it was appropriate to build specific types of temples for specific types of icons.
 Several different shapes may be combined in specific periods and places to create their own unique style.

Q 37.C
 The Indus Valley Civilization was an ancient civilization located in what is Pakistan and northwest India
today, on the fertile flood plain of the Indus River and its vicinity.
 Some of the important India Valley sites can be traced as:
 Banawali - It is an archaeological site belonging to Indus Valley Civilization period in Haryana. Also
referred to as Vanavali, it is on the left banks of dried up Sarasvati River. Hence pair 1 is correctly
matched.
 The excavations have yielded a three-fold culture sequence:
 Pre-Harappan (Early-Harappan)
o Represented by the existence of well-planned houses and a fortification wall made of molded bricks.
o In the pottery of this period, pre-Harappan painted motifs tended to become simpler sparse and use of
white pigment becomes progressively less popular.
 Harappan
o marked by the presence of a well-planned fortified township laid in a radial pattern.
o The sophisticated red ware decorated with animal and floral designs comprises dish-on-stand, 'S'-
shaped jar, perforated jar, vase, cooking handis, beaker, basin and goblet etc.
o A terracotta plough model is important as it is a complete specimen found so far in Harappan Culture.
o Other remarkable finds are beads and bangles of semi-precious stones, terracotta and shell, chert
blades, weights and sportsmen of ivory and bone, beads and foil in gold, charred barley grains etc.
 Bara (post-Harappan)
o It is represented by the most distinguished pottery, which is sturdy and heavy, made of fine clay,
baked carefully and dressed with deep-toned oily gloss.
 Kalibangan - It is an ancient town located on the banks of the Ghaggar river in Rajasthan. It literally
means 'black bangles' which it derives from the dense distribution of the fragments of black bangles which
were found at the surface of its mounds. Hence pair 2 is not correctly matched.
o It has relics of both
• Pre-Harappan settlements from the 3500 BC – 2500 BC
• Harappan settlements from the 2500 BC – 1750 BC, but also the
o The excavation of this site proves that a well - established lifestyle existed before the Harappan
civilization in India.
o It was also discovered that for the ceramic industry, Rajasthan was a significant center.
o The pottery of this place has similar designs with those of the Harappan civilization.
o The excavations at Kalibangan revealed Harappan seals, human skeletons, unknown scripts, stamps,
copper bangles, beads etc.
o This was also the site of discovering the most primitive ploughed field that dated back to 2800 BC.
o In 2600 BC, the first archaeologically recorded earthquake took place at this site, which marked the
end of the Pre-Harappan Civilisation
 Dholavira - It is an archaeological site in Kutch District of Gujarat. Hence pair 3 is correctly matched.
o The city of Dholavira was found to be very well-planned, divided into three stages to form a layered
city with a lower town, a quadrangular middle town and a citadel.
o It was famous for water conservation techniques such as rock-cut wells and is possibly the earliest
example of rock-cut wells.
o One of the unique features of Dholavira is that, unlike other Harappan cities like Harappa and
Mohenjodaro, the city is built almost exclusively of stones instead of bricks.
 Shortughai - It is located in the Amu Darya (Oxus River) valley, in northern Afghanistan. There the
remains of a small Harappan colony, presumably sited so as to provide control of the lapis lazuli export
trade originating in the neighboring city of Badakhshan. Hence pair 4 is correctly matched.

Q 38.D
 The Cholas possessed a large army consisting of soldiers, cavalry, archers, and elephants. They had a very
powerful and well-organised military force. They were one of the earliest kingdoms to have a navy.
 The navy reached to its heights under the Cholas. They took under them the Malabar and Coromandal
coasts. With the expansion of Navy and overseas conquest, the Chola Navy became the strongest force in
the region. It also monopolized the trade and commerce in Bay of Bengal and parts of Indian ocean. Thus,
the Bay of Bengal for sometime became a 'Chola lake'.
 Hence, option (d) is the correct answer.

Q 39.B
• According to Jaina tradition, there are 24 Tirthankaras – those who guide men and women across the river
of existence.
 Rishabhdev/Adinath was the first one and Mahavira was the last.
 About Jain Philosophy & Traditions:
o The basic philosophy of the Jainas was already in existence in north India before the birth of
Vardhamana, who came to be known as Mahavira, in the sixth century BCE. Hence statement 1 is
not correct.
o Asceticism and penance are required to free oneself from the cycle of karma. This can be achieved
only by renouncing the world; therefore, monastic existence is a necessary condition of
salvation. Hence statement 2 is correct.
o According to Jaina teachings, the cycle of birth and rebirth is shaped through karma. Hence
statement 3 is correct.
o The most important idea in Jainism is that the entire world is animated: even stones, rocks and water
have life.
 Jaina monks and nuns take five vows to abstain from:
o injury to others & killing
o stealing
o lying
o possessing property
o and to observe celibacy

Q 40.D
 The period which began in about 200 B.C. did not witness a large empire like that of Mauryas, but it is
notable for intimate contacts between Central Asia and India. A series of invasions began in about 200
B.C. The first to cross the Hindukush were the Greeks, who ruled Bactria, lying south of the Oxus river in
the area covered by north Afganistan. The invaders came one after another, but some of them ruled at one
and the same time. One important cause of invasions was the weakness of the Seleucid empire, which
had been established in Bactria and the adjoining areas of Iran called Parthia.
 On account of growing pressure from the Scythian tribes, the later Greek rulers were unable to
hold their power in this area. With the construction of the Chinese Wall, the Scythians were now
pushed back from the Chinese border. So they turned their attention towards neighbouring Greeks and
Parthians. Pushed by the Scythians tribes, the Bactrian Greeks were forced to invade India. Hence option
(d) is the correct answer.
Q 41.A
 Shivaji's system of administration was largely borrowed from the the Deccani states.
 He designated eight-ministers, sometimes called as Ashtapradhan, it was not in the nature of council of
ministers, each minister being directly responsible to the ruler.
 The Amatya or Majumdar was responsible for the account of income and expenditure of the state.
 The Mantri or Waqia-Nawis was responsible for the personal safety of the king and supervise his daily
routine-work.
 The duty of the Sachiv was to see that all royal letters and dispatches were drafted in the proper style. He
also checked the accounts of the paraganas.
 The Sumant or Dabir was the foreign minister. He advised the king regarding questions of war and peace
with other states, received foreign ambassadors, collected news from other states and advised the king in
appointing ambassadors to foreign states.
 The Senapati or Sar-i-Naubat was in charge of recruitment, organisation, and discipline, training of
soldiers and arranging their supplies.
 Pandit Rao or Danadhyaksha looked after the charitable and religious works of the state. He also tried
to attempt moral uplift of the people.
 The Nyayadhish was next to the king, and was the highest judicial authority in the kingdom.

Q 42.C
 Based upon a distinct structure, shape and size, there are two major
classifications of temple architecture:
o Nagara style in Northern India.
o Dravida Style in Southern India
 Some of the features of Nagara style:
o Entire temple to be built on a stone platform with steps leading up
to it. Hence statement 1 is correct.
o It does not usually have elaborate boundary walls or gateways as in
South India. Hence statement 2 is not correct.
o The garbhagriha is always located directly under the tallest
tower. Hence statement 3 is correct.
 There are many subdivisions of nagara temples depending on the shape
of the shikhara such as:
o Rekha-Prasada
o Phamsana
o Valabhi
Q 43.A
• Statement 1 is correct:The Ahom kingdom (1228–1826, also known as Kingdom of Assam) was a
kingdom originating in Medieval India, in the Brahmaputra Valley in Assam, India. It is
well known for maintaining its sovereignty for nearly 600 years and successfully resisting Mughal
expansion in Northeast India.
 Statement 2 is not correct: In Assam, the Ahom kings had their paiks, people who were obliged
to render military service in exchange for land.
 The Paik System was systematically implemented by Momai Tamuli Barbarua under the patronage of
King Pratap Singha. Under the Paik system, every adult male between the age 16 to 50 was registered as a
paik for State service.
 Each paik in return for his service to the State was allotted two puras of best arable land called ga-mati,
free of charge which was neither hereditary nor transferable.
 The royal services that the paiks tended to were
o defense (the Ahom kingdom did not have a standing army till the beginning of 19th century and its
army consisted of the militia formed of paiks),
o civil construction (embankments,roads, bridges, tanks etc),
o military production (boats, arrows, muskets) etc.

Q 44.A
 The emergence of Virashaiva tradition was witnessed during the twelfth century in Karnataka. It was
founded by a Brahmana named Basavanna who was initially a Jaina and a minister in the court of a
Chalukya king. His followers were known as Virashaivas (heroes of Shiva) or Lingayats (wearers of the
linga). Hence, statement 1 is correct.
 The Lingayats worship Shiva in his manifestation as a linga, and men usually wear a small linga in a
silver case on a loop strung over the left shoulder. Those who are revered include the jangama or
wandering monks. Lingayats believe that on death the devotee will be united with Shiva and will not
return to this world. Therefore they do not practise funerary rites such as cremation, prescribed in the
Dharmashastras. Instead they ceremonially bury their dead.
• The followers of this tradition challenged the idea of caste and the “pollution” attributed to certain groups
by Brahmanas. They also questioned the theory of rebirth. Hence, statement 2 is not correct.

Q 45.D
• Statement 1 is correct: Among the merchant classes, some specialized in wholesale trade, and others in
the retail trade. The wholesale traders were known as ‘seth’ or ‘bohra’ and the retail traders were known
as ‘beoparis’ or ‘banik'.
 The Mughals paid attention to roads and sarais, which made communication easier. A uniform tax was
levied on goods at the point of their entry into the empire.
 Rahdari (a transit duty, a toll) or Road cesses was declared illegal, though it continued to be collected
by some of the local rajas (kings).
 Statement 2 is correct: The Mughals introduced silver rupees of high purity, which became a standard
coin in India and abroad and that helped in the growth of India's trade as well.
• Statement 3 is correct: The ‘sarrafs’ (shroff) specialized in changing money, keeping money in deposit
or lending it, or transmitting it from one part of the country to the other by means of ‘hundi.’
• The ‘hundi’ was a letter of credit payable after a certain period. The use of hundis made it easier to
move goods or to transmit money from one part of the country to another.
 When needed, the hundis were cashed at a discount rate, which sometimes included insurance so that the
cost of goods lost or destroyed in transit could be recovered.

Q 46.A
 The chalcolithic (stone-copper age) phase started after the end of the neolithic phase.
 The later period of this age is characterized by stable and widespread settlements which is called as Jorwe
Culture. It is called so because its type site is provided by Jorwe, a village situated on the Pravara
River. Hence statement 1 is correct.
 It was a nucleated settlement with more than 35 houses of different sizes and shapes. Thus the chalcolithic
economy was a village economy.
 They did not know the art of writing. Indus people were the first in India to develop writing. Hence
statement 2 is not correct.
Q 47.A
• Muslim rulers were theoretically to be guided by the ulama, who were expected to ensure that they ruled
according to the shari‘a. However, the situation was complicated in the subcontinent, where there were
populations that did not subscribe to Islam.
 It is in this context that the category of the zimmi (derived from the Arabic word zimma, protection),
developed for peoples who followed revealed scriptures (like Bible, Quran, Vedas, etc), such as the Jews
and Christians, and lived under Muslim rulership. They paid a tax called jizya and gained the right to be
protected by Muslims. In India this status was extended to Hindus as well.
 Hence, option (a) is the correct answer.

Q 48.B
 Pietra-dura is a Pictorial mosaic work using semi-precious stones. It is commonly found on walls,
cenotaphs and marble jalis in the Taj Mahal.
 Tessellation is the decoration on walls and floors with mosaic.
o Mosaic is a surface decoration made by inlaying small pieces of variously colored material (stones,
glasses, etc) to form pictures or patterns.
 Stucco is a fine plaster used for coating wall surfaces. It can be a smooth paintable surface or a wet
ground for fresco painting.
Q 49.C
 Epigraphic materials of the Gupta period from North Bengal clearly show that Pundravardhana bhukti
was divided into several visayas or districts, which were again subdivided into vithis and mandalas. The
bhukti was entrusted to an Uparika or Uparikamaharaja (provincial governor) appointed directly by the
reigning Gupta emperor. The administrative and territorial jurisdiction of Pundravardhana expanded in the
Pala period.
 Pundravardhanabhukti which covered almost the whole of the north Bengal, now mostly in Bangladesh
has records of land sale documents on copper plates from AD 432-33. In the Gupta period,
Pundravardhana or Paundravardhana became an important bhukti or territorial division. Option
(c) is the correct answer.
 The deltaic portion of Bengal formed by the Brahmaputra was called Samatata, was made to acknowledge
the authority of Samudragupta, it covered south-east Bengal.
 Samatata an ancient territorial unit in south-eastern Bengal, the name of which is descriptive rather
than ethnic. Samatata was a distinct entity, well-known and well-recognized since Samudragupta's time
(4th century AD).
Q 50.D
 Jizya or jizyah is a per capita yearly tax historically levied on non-Muslim subjects, called the dhimma,
permanently residing in Muslim lands governed by Islamic law. The Quran and hadiths mention jizya
without specifying its rate or amount.
 The tax was first introduced in India under the rule of Qutb-ud-din Aibak.
 Aurangzeb again introduced the jizyah (or the poll tax) (it was abolished by Akbar). According to
the sharia, in a Muslim state, the payment of jizyah was obligatory, for the non-Muslims. Later, the tax
was again abolished by Jahandar Shah.
 Aurangzeb, in fact, did not try to change the nature of the state, but reasserted its fundamentally Islamic
character. Aurangzeb's religious beliefs cannot be considered as the basis of his political policies.
 The incidence of jiziya was very minimal; also, women, children, the disabled and the indigent, that is
those whose income was less than subsistence were exempted, as were those in government
services. Hence option (d) is not correct.

Q 51.A
 Statement 1 and 2 are correct: While most mahajanapadas were ruled by kings, some, known as ganas
or sanghas , were oligarchies, where power was shared by a number of men, often collectively called rajas
. Both Mahavira and the Buddha belonged to such ganas. In some instances, as in the case of the Vajji
sangha , the rajas probably controlled resources such as land collectively.
 Statement 3 is not correct: Between the sixth and the fourth centuries BCE , Magadha (in present-day
Bihar) became the most powerful mahajanapada. It was not an oligarchy as Buddhist and Jaina writers
who wrote about Magadha attributed its power to the policies of individuals: ruthlessly ambitious kings of
whom Bimbisara, Ajatasattu and Mahapadma Nanda are the best known, and their ministers, who helped
implement their policies.
Q 52.D
 Gautama Buddha recommended an eightfold path (ashtangika marga) for the elimination of human
misery. This path is attributed to him in a text of about the third century BC.
 It comprised right observation, right determination, right speech, right action, right livelihood, right
effort, right awareness, and right concern. If a person follows this eightfold path, he would free himself
from the machinations of priests, and would reach his destination.

Q 53.A
 Statement 1 is correct: The Pala empire dominated eastern India till the middle of the ninth century. It
was founded by Gopala, probably in 750 A.D when he was elected King by the notable men of the area to
end the anarchy prevailing there. He was succeeded by Dharmapala.
 Statement 2 is not correct: Pala rulers were great patrons of Buddhist learning and religion. The Nalanda
University which had been famous all over the eastern world was revived by Dharmapala and 200 villages
were set apart for meeting its expenses. He also founded Vikramsila University which became second
only to Nalanda in fame. It was located on the top of a hill, on the banks of the Ganga in Magadha amidst
pleasant surroundings. The Nalanda University was founded by Gupta emperor Kumargupta I.
 Statement 3 is not correct: The Palas had close trade contacts and cultural links with South-East Asia.
The trade with South-East Asia was very profitable and added greatly to the prosperity of the Pala empire.

Q 54.A
 Some of the major Buddhist Texts are:
o Dipavamsa (History of the Island) is the oldest extant historical record of Sri Lanka, compiled in the
4th century
o Therigatha - It is an anthology of poems in the Pali language by and about the first Buddhist women.
It was composed more than two millennia ago.
o Ashokavadana - It is a Buddhist text related to the legend of the Maurya Emperor Ashoka. It was
written in the 2nd century CE.
 The Ashokavadana is an account of the birth and reign of King Ashoka.
 According to the legend, the birth of Ashoka was predicted by the Buddha, in the story, "The Gift of
Dust":
o Vessantara Jataka - Vessantara was a very generous king who, in Buddhist mythology, is considered
to be a previous incarnation of the Buddha Gotama.
 Uttaradhyayana Sutta is a Jain text.
o It is one of the four 'Mulasutras' of the canons of Shvetambar Jain.
o The main text is written in Ardhamagadhi Prakrit script accompanied by a Sanskrit commentary.

Q 55.B
 Statement 2 is not correct: Varna, according to Hindu scriptures, refers to the classification of people
based on their qualities. The term is derived from the Sanskrit word, vr, which means "to describe," "to
classify" or "to cover." Varna is classified into four types: brahman, kshatriya, vaishya and shudra.
According to the Brahmanical texts, another criterion apart from gender for regulating access to wealth
was varna.
 Statement 1 is correct: This Brahmanical view of society was codified in the Dharmasutras and
Dharmashastras, other traditions developed critiques of the varna order like the Buddhists recognized that
there were differences in society, but did not regard these as natural or inflexible.
• Statement 3 is not correct : Untouchables were placed outside the Varna framework. Beyond the four
Varnas, the Brahmanas considered some people as being outside the varna system, they developed a
sharper social divide by classifying certain social categories as “untouchable” This rested on a notion that
certain activities, especially those connected with the performance of rituals, were sacred and by extension
“pure”. Those who considered themselves pure avoided taking food from those they designated as
“untouchable”.

Q 56.D
 The mural is a painting applied to and made integral with the surface of a wall or ceiling.
 Some of the traditional forms of murals are:
o Pithoro in parts of Madhya Pradesh, Rajasthan and Gujarat: They are done by a group of
painters called Lakharas. These paintings are noted for ritual characteristics. The protagonists of the
entire painting are horses of the gods and goddesses and ancestors in vibrant colours. The painting is
adorned with motifs from nature, daily human activities, animals, farming, trade, important members
of the community, along with several new elements, which symbolically represent modernity.
o Mithila painting in northern Bihar’s Mithila region: The themes in Mithila painting (also known
as Madhubani Painting) revolve around Hindu deities like Krishna, Rama, Lakshmi, Shiva, Durga and
Saraswati. The natural themes that are used include the Sun, the Moon and the religious plants like
tulsi.
 The painting was traditionally done by womenfolk on freshly plastered mud walls and floors of huts, but
now they are also done on cloth, handmade paper and canvas.
 One can also find paintings based on scenes from the royal courts and social events, like weddings.
 If any empty space is left after painting the main theme, it is filled up with the motifs of flowers, animals
and birds or geometric designs.
 Mithila painting, as a domestic ritual activity, was unknown to the outside world until the massive India-
Nepal border earthquake of 1934 when the houses and walls tumbled down. Then British colonial officer
in Madhubani District, William G. Archer, while inspecting the damage 'discovered' the paintings on the
newly exposed interior walls of Mithila homes.
o Warli paintings in Maharashtra:
 Warli is the name of the largest tribe found on the northern outskirts of Mumbai, in Western India.
 Warli tribals decorate their house walls with paintings depicting their lives: planting saplings, carrying
grain, dancing, travelling to market and other routine activities of their daily lives.
 They do not depict mythological characters or images of deities
 Its roots go back to the ancient times - cave paintings (murals) in rock shelters at Bhimbetka, Madhya
Pradesh.
 The unique feature of these paintings is that mainly white colour is used to paint. This colour is prepared
by mixing rice paste and water with the addition of gum for adhesiveness. The background is prepared by
using mud and cow dung (if done on the walls of the hut). The figures reflect the naturalism.
 Hence only pairs 2 and 3 are correctly matched.

Q 57.C
 Humayun Nama has been written by Gulbadan Begum in 1587.
 Akbar asked his aunt to write whatever she remembered about her brother's life. Gulbadan Begum took
the challenge and produced a document which came to be known as Humayun-nama.
 Alamgir Nama has been written by Muhammad Kazim in 1668.
 It is the history of up to the tenth year of the reign of Aurangzeb (1668), but the emperor forbade him to
continue the work after this period. The history was a voluminous work consisting of 1107 printed pages
(published by the Calcutta Asiatic Society of Bengal in 1868).
 Badshah Nama has been written by Abdul Hamid Lahori in 1639-47.
 Abdul Hamid Lahori was a traveller and historian during the period of Mughal Emperor Shah Jahan who
later became a court historian of Shah Jahan (Shah Jahan's official chronicler).
 The book is also known as Padshahnama, it is about the reign of Shah Jahan.
Other Books
• In 1589, Babur’s memoirs translated into Persian as Babur Nama.
• In 1589-1602, Abu’l Fazl works on the Akbar Nama.
 In 1605-22, Jahangir writes his memoirs, the Jahangir Nama.

Q 58.A
 Statement 1 is correct: In Delhi Sultanate, Sultan was the commander-in-chief of the army. Diwan-I-
Ariz was established by Balban to assist the Sultan in Military administration.
• Statement 2 is correct: Diwan-i-Kohi was the Agriculture department established by Mohammed-
Bin-Tughlaq as a part of Agricultural reforms.The main object of the department was to increase the
land under cultivation, and to distribute loans to the peasants. He established a “Famine Code” to relieve
the victims of Famine.
 Statement 3 is not correct: Diwan-i-Risalat was for dealing with religious affairs and was also in charge
of the grant of stipends to scholars and pious men. However, among historians there is one more view that
he was minister for foreign affairs and was in charge of diplomatic correspondence. There was no specific
department for industrial development.
 Important departments in Delhi Sultanate
o Diwan-i-Waziarat – Prime Minister and Finance Minister
o Diwani-i-Risalat – Religious Affairs Department
o Sadr-us-Suddar – Department of Islamic Law
o Diwan-i-lnsha – Correspondence Department
o Diwan-I-Ariz – Military administration
o Qazi-ul-quzar – Department of law/Justice.
o Diwan-i-Bandgan - affairs of the slaves(by Feroz Shah Tughlaq)
o Diwan-i- Mustakhara- Department of revenue

Q 59.D
 Jehangir was succeeded by his second son Shah Jahan in 1628. The Mughal Empire was at its zenith
during Shah Jahan's rule.
 In 1639, he shifted his capital from Agra to Delhi and called the new capital as Shahjahanabad. It
was a planned city.At the centre of this settlement was Qila-i-Mubarak, the palace-fortress. The city
was encircled with a wall with 14 gates, from where Shahjahanabad gets its sobriquet of Walled
City.
 Monthly scale (Month-ratio or Month-scale system) was introduced in Mansabdari system by
Shahjahan.
 It was a new scaling device under which the salaries of mansabdars were put on a month scale: ten
months, eight months, six months or even less.
 The obligations of the mansabdars for maintaining a quota of sawars were brought down accordingly.
 The month-scale system was applied to both jagirs and those who were paid in cash.
 Francois Bernier was closely associated with the Mughal court, as a physician to Prince Dara Shukoh,
the eldest son of Emperor Shah Jahan, and later as an intellectual and scientist, with Danishmand Khan,
an Armenian noble at the Mughal court. He was witness to the fierce war of succession among the sons of
Shah Jahan.
 Jats and Satnamis raised the banner of rebellion during the reign of Aurangzeb.

Q 60.C
 Amoghavarsha was a Rashtrakuta emperor. He is regarded as one of the greatest ruler of the
Rashtrakuta dynasty, and one of the great emperors of India. He ruled for a reign of 68 years
and preferred the pursuit of religion and literature to war.
 Many Kannada and Sanskrit scholars prospered during his rule, including the great Indian mathematician
Mahaviracharya who wrote Ganita-sara-samgraha, Jinasena, Virasena, Shakatayan and Sri Vijaya .
 Amoghavarsha himself was an accomplished poet and scholar. He wrote the Kavirajamarga, the
earliest poetics work in Kannada and Prashnottara Ratnamalika, a religious work in Sanskrit.
 He moved the Rashtrakuta regal capital from Mayurkhandi in the Bidar district to Manyakhet in the
Gulbarga district in the modern Karnataka state. He is said to have built the capital city Manakhet to
"match that of Lord Indra". The capital city was planned to include elaborately designed buildings for
the royalty using the finest of workmanship.
 Hence, option (c) is the correct answer.

Q 61.C
 The techniques of movement in Odissi are built around the two basic postures of the Chowk and the
Tribhanga. The chowk is a position imitating a square - a very masculine stance with the weight of
the body equally balanced. The tribhanga is a very feminine stance where the body is deflected at the
neck, torso and the knees.
 Hence option (c) is the correct answer.

Q 62.A
 Jatisvaram is a vocal form in Carnatic music tradition. It is very similar to the svarajati in musical
structure but it has no sahitya or words. The piece is sung with solfa syllables only. It is noted for its
rhythmical excellence and the jati pattern used in it. This is a musical form belonging to the realm of
dance music. In some Jatisvarams, the Pallavi and Anupallavi are sung to jatis and the Charanas are sung
to a mixture of svaras and jatis. There are also Ragamalika Jatisvarams. Hence option (a) is the correct
answer.
Q 63.A
• Statement 1 is correct: Akbar organized and strengthened his army and encouraged the mansabdari
system. “Mansab” is an Arabic word, which means ‘rank’ or ‘position'.
 Under the mansabdari system, every officer was assigned a rank (mansab). The lowest rank was 10, and
the highest was 5,000 for the nobles; however, towards the end of the reign, it was raised to 7,000. Princes
of the blood received higher mansabs.
 The mansabs (ranks) were categorized as Zat and Sawar.
• The word ‘zat’ means personal. It fixed the personal status of a person, and also his salary.
• The ‘sawar’ rank indicated the number of cavalrymen (sawars) a person was required to maintain.
 Statement 2 is not correct: The Mughal mansabdars were paid very handsomely; in fact, their salaries
were probably the highest in the world at the time.
• A mansabdar, holding the rank of −
o 100 zat, received a monthly salary of Rs. 500/month.
o 1,000 zat received Rs. 4,400/month.
o 5,000 zat received Rs. 30,000/month.
 Mansabdars were also paid by assigning them jagirs.
 Statement 3 is not correct; Out of his personal pay, the mansabdar was expected to maintain a corps
of elephants, camels, mules, and carts, which were necessary for the transport of the army.

Q 64.B
• The Buddha had two kinds of disciples – lay worshippers (upasakas) and monks (bhikkus or shramanas).
The latter were organized into the Sangha or congregation. The membership of the Sangha was open to
all, male or female above twenty years of age and who were free from leprosy, consumption and
other infections diseases. However, slaves, soldiers and debtors were not allowed to enter the
Sangha without the permission of their superior. As they lived on alms, they were known as
bhikkhus.
 Sangha was an organization of monks who later on also became teachers of dhamma. The rules of
Buddhist sangha were contained in Vinaya Pitaka.
 Some of the features of sanghas were:
o Initially, only men were allowed into the sangha, but later women also came to be admitted. Hence
statement 1 is not correct.
o All members of sangha were regarded as equal, having shed their earlier social identities on becoming
bhikkhus and bhikkhunis. Hence statement 2 is correct.
o The internal functioning of the sangha was based on the traditions of ganas and sanghas, where
consensus was arrived at through discussions. If that failed, decisions were taken by a vote on the
subject. Hence statement 3 is not correct.

Q 65.A

 Tamasha is a traditional form of Marathi theatre, widely performed by local or travelling theatre groups
within the state of Maharashtra. It is often associated with singing and dancing. It has evolved from folk
forms such as Gondhal, Jagran and Kirtan. In Tamaasha, the female actress is the chief exponent of dance
movements in the play. She is known as Murki. Classical music, footwork at lightning-speed, and vivid
gestures make it possible to portray all the emotions through dance. In Maharashtra, the Kolhati groups
are traditionally associated with the performance of Tamasha. Hence pair 1 is correctly matched.
 Therukoothu is a popular form of folk drama of Tamil Nadu. It literally means "street play". It is mostly
performed at the time of annual temple festivals of Mariamman (Rain goddess) to achieve rich
harvest. At the core of the extensive repertoire of Therukoothu there is a cycle of eight plays based on the
life of Draupadi. Kattiakaran, the Sutradhara of the Therukoothu performance, gives the gist of the play to
the audience and Komali entertains the audience with his buffoonery. Hence pair 2 is not correctly
matched.
 Bhaona is a traditional form of entertainment, always with religious messages, prevalent in Assam. In
Bhaona cultural glimpses of Assam, Bengal Orissa, Mathura and Brindavan can be seen. The
Sutradhaar, or narrator begins the story, first in Sanskrit and then in either Brajboli or Assamese. Hence
pair 3 is not correctly matched.
Q 66.D
• The spread of Jainism in Karnataka is attributed to Chandragupta Maurya (322– 298 BC). The
emperor became a Jaina, gave up his throne, and spent the last years of his life in Karnataka as a Jaina
ascetic.
 When Bindusara became an adult, Chandragupta Maurya decided to pass on the baton to his only son
Bindusara. After making him the new emperor, he requested Chanakya to continue his services as the
chief advisor of the Maurya dynasty and left Patliputra. He renounced all worldly pleasures and
became a monk as per the tradition of Jainism. He traveled far into the south of India before
settling down in Shravanabelagola .
 Around 297 BC, under the guidance of his spiritual guru Saint Bhadrabahu, Chandragupta
Maurya decided to give up his mortal body through Sallekhana. Hence he started fasting and on one fine
day inside a cave at Shravanabelagola, he breathed his last, ending his days of self-starvation.

Q 67.B
 The city of Mandu is located near Indore overlooking the Malwa Plateau to the north and the Narmada
valley to the south.
 Mandu is a typical representation of the medieval provincial style of art and architecture.
 The structures are designed in the style of arched pavilions, light and airy so that these buildings did not
retain heat.
 The Royal Enclave located in the city comprised a cluster of palaces and other structures such as:
o Hindola Mahal was the audience hall of the Sultan and also the place where he showed himself to his
subjects. Hence option 1 is correct.
o Jama Masjid of Mandu was built to accommodate many worshippers for Friday prayers.
o Jahaaz Mahal is a two-storey ‘ship palace’ built by Sultan Ghiyasuddin Khilji. Hence option 3 is
correct.
 Gol Gumbad
o It is situated in Bijapur District of Karnataka. Hence option 2 is not correct.
o It was built by Muhammad Adil Shah (1626–1656), the seventh Sultan of the Adil Shahi Dynasty of
Bijapur (1489–1686).
Q 68.C
 Option (c) is correct: The Shakas who came from Central Asia, were regarded as mlechchhas ,
barbarians or outsiders by the Brahmanas.
 Powerful mlechchhas were familiar with Sanskritic traditions.
 Rudradaman, the best-known Shaka ruler( c. second century CE ), rebuilt Sudarshana lake.
 People living in forests were said nishadas, goldsmiths as suvarnakara. Whereas, Sanskrit texts and
inscriptions used the term vanik or sarthavaha to designate merchants.

Q 69.B
 The terms for war in the Rig Veda is gavishthi or search for cows, and cow seems to have been the most
important form of wealth. Whenever we hear of gifts made to priests, they usually consist of cows and
women slaves and never of land. The Rig Vedic people may have occasionally occupied pieces of land
for grazing, cultivation, and settlement, but land did not form a well-established type of
private property. Hence statement 1 is not correct.
 The Rig Vedic people had a superior knowledge of agriculture. The ploughshare is mentioned in the
earliest part of the Rig Veda, though some consider it to be an interpolation, and was possibly made of
wood. They were acquainted with sowing, harvesting, and threshing, and knew about the different
seasons. Agriculture was also well known to the pre-Aryans who lived in the area associated with the
Vedic people, but was perhaps used primarily to produce fodder. Hence statement 2 is correct.
 The Rig Veda mentions such artisans as the carpenter, chariot-maker, weaver, leather worker, and potter.
This indicates that they practised all these crafts. The term ayas, used for copper or bronze, shows that
metalworking was known. Hence statement 3 is correct.

Q 70.D
 Trade, local and long-distance constituted a very important source of royal revenue. The tip of the
peninsula and adjacent regions were very fertile. The land produced paddy, ragi and sugarcane. In
addition to this Tamil region produced pepper, grains, fruit and turmeric.
 The southern states profited immensely from their natural resources and foreign trade. They grew spices,
especially pepper, which was in great demand in the western world. Their elephants supplied ivory,
which was highly valued in the West.
 The sea yielded pearls and their mines produced precious stones, and both these were sent to the
West in good quantities.
 In addition to this, they produced muslin and silk. A cotton cloth as thin as the slough of a snake was
also produced. The early Tamil poems also mention weaving o complex patterns on silk. Uraiyur was
noted for its cotton trade.
 Hence all the options are correct.

Q 71.A
• Statement 1 is correct; Kabul and Qandhar had always acted as staging places for an invasion in India,
Babur’s advent made Kabul and Qandahar the integral parts of an empire comprising north India.
 Babur and his successors strengthen India's security from an external invasion, which were persistent
from the last 200 years.
 Geographically Kabul and Qandhar positioned in the trade route; therefore, the control of these two
regions strengthened India's foreign trade.
 Statement 2 is correct; Babur introduced a new concept of the state.
o The strength and prestige of the Crown.
o The absence of religious and sectarian bigotry.
o The careful fostering of culture and the fine arts with the amalgamation of Persian and Indian
themes.
o Char bagh style of architecture came into existence.
 Statement 3 is not correct; Gunpowder was known in India before Babur. Gunpowder is the first
discovery of explosive material. It is said to arrive in India in mid of 14th century. It was used in the
second half of the 14th-century war by Bahmani kingdom as well as Delhi Sultanate. Babur used it in
Battle of Panipat in 1526.

Q 72.B
 Manipuri dance is associated with rituals and traditional festivals. It is particularly known for its Hindu
Vaishnavism themes, and exquisite performances of love-inspired dance drama of Radha-Krishna
called Raslila. However, the dance is also performed to themes related to Shaivism, Shaktism and
regional deities such as Umang Lai during Lai Haraoba.
 Manipur dance has a large repertoire, however, the most popular forms are the Ras, the Sankirtana and
the Thang-Ta.
 Ras: Ras dance is part of the traditional story of Krishna described in Hindu scriptures such as the
Bhagavata Purana and literature such as the Gita Govinda, where he dances with Radha and her sakhis.
There are five principal Ras dances of which four are linked with specific seasons, while the fifth can be
presented at any time of the year. In Manipuri Ras, the main characters are Radha, Krishna and the gopis.
The Ras costume consists of a richly embroidered stiff skirt which extends to the feet.
 Sankirtana: The Kirtan form of congregational singing accompanies the dance which is known as
Sankirtana in Manipur. The male dancers play the Pung and Kartal while dancing. The masculine aspect
of dance - the Choloms are a part of the Sankirtana tradition. The Pung and Kartal choloms are
performed at all social and religious festivals.
 Thang-Ta: The Art of the Sword and Spear is the traditional martial art of Manipur in Northeast India.
It is part of the great heroic tradition of Manipur.
 Kalasam is a part of repertoire of Kathakali dance. These are pure dance sequences where the actor is
at great liberty to express himself and display his skills. The leaps, quick turns, jumps and the rhythmic
co-ordination make kalasams, a joy to watch.

Q 73.C
 Pair 1 is correctly matched: Wali was a friend of God was a sufi who claimed proximity to Allah,
acquiring his grace to perform miracles (karamat).
• Pair 2 is not correctly matched: The word used for Sufism in Islamic texts is tasawwuf. It deals with the
system of morality. Its function is to purify the heart from the lowly bestial attributes of lust, anger,
malice, jealousy, love of fame, etc. It is therefore man’s direct relationship with his maker which is the
breath and life of religion, and it is the study and cultivation of this relationship that the word tasawwuf
connotes.
 Pair 3 is correctly matched: Ziyarat refers to the practice of pilgrimage or sufi saints' grave, particularly
on his death anniversary. This signified the union of his soul with God. This was because people believed
that in death saints were united with God, and were thus closer to Him than when living.
Q 74.B
 Vajrayana is a form of Buddhism probably originated from the Mahayana tradition. It is based on a
complex philosophical and ritual system meant to provide a path towards enlightenment. The term
Vajrayana roughly means ''the way of the diamond.'' The word Vajra refers to the diamond-hard
thunderbolt that was used as a weapon by the Hindu god of thunder and rain, Indra. Yana refers to the way
or the spiritual vehicle for achieving enlightenment.
 Vajrayana Buddhism developed in India around the 6th or 7th century CE. By the seventh century
CE, the Buddhist monasteries had come to be dominated by ease-loving people and became centres of
corrupt practices which had been prohibited by Gautama Buddha. The new form of Buddhism was
known as Vajrayana. Hence statement 1 is not correct.
 Tantric ideas began to be incorporated into the teachings of Buddhist universities in Northern
India, gradually leading to the development of this new tradition. Vajrayana Buddhism is full of
symbolism and rites, mostly focused on the search for enlightenment. The rites are often meditation that
includes tantric practices, combining the physical and spiritual worlds. Hence statement 2 is correct.

Q 75.A
 The Indo-Iranian contact lasted for about 200 years. It gave an impetus to Indo- Iranian trade and
commerce. The cultural results were more significant. Iranian scribes brought into India a form of
writing that came to be known as the Kharoshthi script. Hence statement 1 is correct.
 The Achaemenian rulers of Iran (Bactrians), who expanded their empire at the same time as the
Magadhan princes, took advantage of the political disunity on the northwest frontier. The Iranian
ruler Darius penetrated north-west India in 516 BC and annexed the Punjab, west of the Indus, and
Sindh.
 It was written from right to left like the Arabic. Some Ashokan inscriptions in north-west India were
written in the third century BC in this script, which continued to be used in India till the third century
AD. Hence statement 2 is not correct.

Q 76.B
• Rashtrakutas were of Kannada origin. Dantivarman or Dantidurga (735 – 756) was the founder of the
Rashtrakutas dynasty. The greatest king of the Rashtrakuta dynasty was Amoghavarsha I . Amoghavarsha
I set up a new capital at Manyakheta (now Malkhed in Karnataka State). Amoghavarsha I was a great
patron of education and literature. Amoghavarsha was converted into Jainism by Jinasena, a Jaina monk.
Amoghavarsha ruled for 63 years. His work Kavirajamarga is the earliest available work on rhetoric,
poetics and grammar in the Kannada language.
 Statement 1 is not correct: The Rastrakutas administration was quite decentralized. The system of
administration was modelled on ideas and practices of Gupta administration. The empire consisted
of directly administered areas and area ruled by vassal chiefs. Vassals were autonomous in their internal
affairs and had a general obligation of loyalty and paying a tribute.
 Directly administered areas were divided into rashtras (provinces) -controlled by rashtrapatis. Rashtras
divided into vishayas or districts governed by vishayapatis. The subdivision was bhukti consisting of 50 to
70 villages under the control of bhogapatis. Village headmen carried on village administration. Village
assemblies played a significant role in the village administration.
 Statement 2 is correct: The Kailash temple at Ellora Caves built by Rastrakuta King Krishna I in
AD 760 is the largest monolithic structure in the world. The Kailasa temple (Cave 16) is one of the 34
caves in Ellora. The temple architecture shows traces of Pallava and Chalukya styles. It appears to be
based on the Virupaksha Temple at Pattadakal and the Kailasa temple at Kanchi, but it is not an exact
imitation of these two temples
 Statement 3 is not correct: Rashtrakuta rulers were tolerant in their religious views and patronised
not only Shaivism and Vaishnavism but Jainism as well. Amoghavarsha was a Jain but he also
patronised other faiths. The rashtrakutas allowed Muslim traders to settle and permitted Islam to be
preached in their dominions. This tolerant policy helped to promote foreign trade which enriched the
Rashtrakutas.

Q 77.B
 The period between 800-900 AD was a period of stagnation and even of decline.This is seen in steady
decline of towns, and the absence of gold and silver coins between the 7th and 10th centuries.
 Statement 1 is not correct: One of the major reasons for the decline was decline in the west of the
Roman empire with which India had a flourishing and profitable trade.
 Statement 2 is correct: Another major reason for the decline of trade was the rise of many states and the
growth of 'localism'. In these states, there was marked growth of self-sufficiency which grew in
conjunction with small towns. Within these states, in many areas, there was a growth of agriculture.
 Statement 3 is correct: The religious prohibitions on travel also played a major role in the decline in
trade and commerce. In some of the Dharamshastras which were written during this period, a ban is put on
travel beyond the areas where the munja grass doesn't grow or where the black gazelle doesn't roam, that
is, outside India. Travel across the salt seas was considered polluting.

Q 78.C
• Many early sculptors did not show the Buddha in human form – instead, they showed his presence
through symbols.
o The empty seat was meant to indicate the meditation of the Buddha. Hence pair 1 is not correctly
matched.
o The stupa was meant to represent the mahaparinibbana. Hence pair 2 is not correctly matched.
o Wheel stood for the first sermon of the Buddha, delivered at Sarnath. Hence pair 3 is correctly
matched.

Q 79.A
 Timeline of Major Religious Developments in Indian History can be categorized as follows:
o 1500-1000 BCE Early Vedic age
o 1000-500 BCE Later Vedic traditions
o Sixth century BCE Early Upanishads; Jainism, Buddhism
o Third century BCE First stupas
o Second century BCE onwards Development of Mahayana Buddhism, Vaishnavism, Shaivism and
goddess cults
o Third century CE Earliest temples
Q 80.B
 Al-Biruni from Uzbekistan visited India during the tenth-eleventh century. His book Kitab-ul-Hind,
written in Arabic, is simple and lucid. It is a voluminous text, divided into 80 chapters on subjects such
as religion and philosophy, festivals, astronomy, alchemy, manners and customs, social life, weights and
measures, iconography, laws and metrology.
 Generally (though not always), Al-Biruni adopted a distinctive structure in each chapter, beginning
with a question, following this up with a description based on Sanskritic traditions, and
concluding with a comparison with other cultures. Some present-day scholars have argued that this
almost geometric structure, remarkable for its precision and predictability, owed much to his
mathematical orientation. Hence, option (b) is the correct answer.
 Rihla
o It is an account of travels of Ibn Battuta who visited India during the fourteenth century. It is written
in Arabic, and provides extremely rich and interesting details about the social and cultural life in the
subcontinent in the fourteenth century.
 Al-Tafhim
o It is a Persian language work by the renowned Al-Biruni containing questions and answers in a format
easily understandable by new learners in sciences. This is the oldest Persian text on Mathematics and
Astrology and was composed simultaneously in Persian and Arabic by Al-Biruni himself.
 Kashful-Mahjub
o It was a book written in Persian by Abu’l Hasan al Hujwiri, who was a sufi saint and settled in
Lahore. The book aims to explain the meaning of Sufism.

Q 81.D
 Historians of religion have divided bhakti movement into two broad categories - saguna (with attributes)
and nirguna (without attributes).
 The former included traditions that focused on the worship of specific deities such as Shiva, Vishnu and
his avatars (incarnations) and forms of the goddess or Devi, all often conceptualised in anthropomorphic
forms. Nirguna bhakti on the other hand was worship of an abstract form of god. Hence, statement 1 is
not correct.
 Nirguna is the concept of a formless God, which has no attributes or quality. Saguna has form, attributes
and quality. While nirguni is knowledge focussed, saguna is love-focused. Thus, the nirguna poetry is
Gyanshrayi (has roots in knowledge) while saguna poetry is premashrayi (has roots in love).
 The most common examples of saguna bhakti include Ramananda and Chaitanya who espoused the
doctrine of incarnation and worshipped the saguna Ram and Krishna respectively. On the other hand, the
most common examples of nirguna bhakti include Kabir, Guru Nanak and Dadu who created a religious
school which rejected the scriptural authority and every form of idol worship. They fought against social
discrimination and strove for Hindu-Muslim reconciliation. Hence, statement 2 is not correct.

Q 82.A
 Statement 1 is correct; After the break-up of the Bahmani kingdom, three powerful states,
Ahmadnagar, Bijapur, and Golconda emerged as the independent states. In 1565, all these three states
united to crush Vijayanagara Empire at the battle of Bannihatti, nearTallikota (also known as battle
of Talikota/Rakshasa Tangadi).
 At the time of Battle of Talikota, Sadasiva Raya was the ruler of Vijaynagar kingdom. But he was a
puppet ruler. The real power was exercised by his minister Rama Raya. He tried to crush the power of
Deccan sultanates by creating difference amongst them.
 However, the Deccan sultanates got a hint of his plan and formed an alliance against Vijayanagar
kingdom. Soon, they attacked the kingdom and looted and destroyed everything after entering the capital.
 Statement 2 is not correct: The glory of last Hindu empire i.e., Vijayanagar Kingdom came to an end
and this gave impetus to Mughals to extend their rule in Deccan and not the Bahmani Kingdom.
 The Kingdom of Mysore, Nayakas of Vellore, Nayakas of Keladi in Shimoga declared their
independence from Vijayanagar.
 The Muslim Sultanates of Deccan could not gain much out of this victory because they soon engaged
themselves in fighting among themselves and fell easy prey to the Mughals.

Q 83.B
• Dashavatar is a religious folk theatre form of southern Maharashtra and northern Goa. It is at least
500 years old. It’s based on mythological tales about Lord Vishnu’s ten incarnations – Matsya (the
fish), Kurma (the tortoise), Varaha (the boar), Narasimha (the lion-man), Vamana (the dwarf),
Parashurama, Rama, Krishna (or Balarama), Buddha and Kalki. The plays are almost entirely improvised,
several hours long and usually performed by seasonal farmers and labourers. Traditionally, they have
been performed inside temples and only by men. Apart from stylized make-up, the Dashavatar
performers wear masks of wood and papier mache.

Q 84.D
 Statement 1 is correct: The rulers of Vijayanagara empire encircled not only the city but also its
agricultural hinterland and forests. The outermost wall linked the hills surrounding the city. The massive
masonry construction was slightly tapered. No mortar or cementing agent was employed anywhere in the
construction. The stone blocks were wedge-shaped, which held them in place, and the inner portion of the
walls was of earth packed with rubble. Square or rectangular bastions projected outwards.
 Statement 2 is correct: A characteristic feature of the temple complexes during the Vijaynagara empire
is the chariot streets that extended from the temple gopuram in a straight line. These streets were paved
with stone slabs and lined with pillared pavilions in which merchants set up their shops.
• Statement 3 is correct: The king’s palace of the Vijaynagara empire is associated with the presence of
two impressive platforms, usually called the “audience hall” and the “mahanavami dibba”. The entire
complex is surrounded by high double walls with a street running between them. The audience hall is a
high platform with slots for wooden pillars at close and regular intervals. Located on one of the highest
points in the city, the “mahanavami dibba” is a massive platform rising from a base of about 11,000 sq. ft
to a height of 40 ft. There is evidence that it supported a wooden structure. The base of the platform is
covered with relief carvings. Rituals associated with the structure probably coincided with Mahanavami
(literally, the great ninth day) of the ten-day Hindu festival during the autumn months of September and
October.

Q 85.C
• Statement 1 is correct; The basic unit of agricultural society was the village, inhabited by peasants
who performed the manifold seasonal tasks that made up agricultural production throughout the year –
tilling the soil, sowing seeds, harvesting the crop when it was ripe.
 During the sixteenth and seventeenth centuries, about 85 per cent of the population of India lived in its
villages. Both peasants and landed elites were involved in agricultural production and claimed rights to a
share of the produce.
 This created relationships of cooperation, competition and conflict among them. The sum of these
agrarian relationships made up rural society. At the same time agencies from outside also entered into
the rural world.
 Most important among these was the Mughal state, which derived the bulk of its income from
agricultural production. Many crops were cash crops that expanded India's external trade.
• Statement 2 is correct; Along with Agricultural activities, the Village peasants were also artisans –
potters, blacksmiths, carpenters, barbers, even goldsmiths – provided specialised services in return for
which they were compensated by villagers by a variety of means.
 The most common way of doing so was by giving them a share of the harvest, or an allotment of land,
perhaps cultivable wastes, which was likely to be decided by the panchayat.
 Another variant of this was a system where artisans and individual peasant households entered into a
mutually negotiated system of remuneration, most of the time goods for services.
• For example, eighteenth-century records tell us of zamindars in Bengal who remunerated blacksmiths,
carpenters, even goldsmiths for their work by paying them “a small daily allowance and diet money”
or services rendered by the servicing castes are paid in cash or in kind. This later came to be described
as the Jajmani System.

Q 86.C
 Traces of early megalithic life appear in the Sangam texts,. The earliest megalithic people seem to be
primarily pastoralists, hunterns and fishermen although they also produced rice. Hoes, sickles and other
iron objects such as wedges, flat celts, arrowheads, long sword and lances, spikes and spearheads occur at
many sites.
 The Sanagm texts suggest war booty was an important source of livelihood. They also state that
when a hero dies, he is reduced to a piece of stone. This reminds of the circles of stone which were
raised on the graves of the meglithic people. It may have led to the later practice of raising hero
stones called virarkal in honour of the heroes who died fighting for the kine (a group of cows) and
other objects. Hence option (c) is the correct answer.

Q 87.B
 Shadow puppets are flat figures. They are cut out of leather, which has been treated to make it
translucent. Shadow puppets are pressed against the screen with a strong source of light behind it. The
manipulation between the light and the screen make colourful shadows.
 Tholu Bommalata: Tholu Bommalata, Andhra Pradesh's shadow theatre has the richest and strongest
tradition. The puppets are large in size and have jointed waist, shoulders, elbows and knees. They are
coloured on both sides. Hence, these puppets throw coloured shadows on the screen. The music is
dominantly influenced by the classical music of the region and the theme of the puppet plays are drawn
from the Ramayana, Mahabharata and Puranas. Hence option b is the correct answer.
 Ravanachhaya: It is Orissa's popular shadow puppet form. The puppets are in one piece and have no
joints. They are not coloured, hence throw opaque shadows on the screen. The manipulation requires great
dexterity since there are no joints. The puppets are made of deerskin and are conceived in bold
dramatic poses. Apart from human and animal characters, many props such as trees, mountains, chariots,
etc. are also used. Although Ravanachhaya puppets are smaller in size-the largest not more than two feet
have no jointed limbs, they create very sensitive and lyrical shadows.
 Togalu Gombayetta: The shadow theatre of Karnataka is known as Togalu Gombeyatta. These puppets
are mostly small in size. The puppets, however, differ in size according to their social status, for instance,
large size for kings and religious characters and smaller size for common people or servants.
 Yampuri: The traditional Rod puppet of Bihar is known as Yampuri. These puppets are made of
wood. Unlike the traditional Rod puppets of West Bengal and Orissa, these puppets are in one piece and
have no joints. As these puppets have no joints, the manipulation is different from other Rod puppets and
requires greater dexterity.

Q 88.B
 Qutab Minar
o It is a soaring, 73 m-high tower of victory, built in 1193 by Qutab-ud-din Aibak immediately after the
defeat of Delhi's last Hindu kingdom.
o The tower has five distinct storeys:
▪ The first three storeys are made of red sandstone
▪ Fourth and fifth storeys are of marble and sandstone. Hence statement 2 is correct.
 The origins of Qutab Minar are shrouded in controversy:
Its construction was started by Qutab-ud-din Aibak in 1192 AD.
o
Some believe it was erected as a tower of victory to signify the beginning of the Muslim rule in India.
o
Others say it served as a minaret to the muezzins to call the faithful to prayer.
o
But it was not built in memory of Sufi saint Khwaja Qutbuddin Bakhtiyar Kaki (born 1173-died 1235)
o
rather was named after him. Hence statement 1 is not correct.
 At the foot of the tower is the Quwwat-ul-Islam Mosque, the first mosque to be built in India.
 Construction:
▪ Qutab-ud-din Aibak, the first Muslim ruler of Delhi, commenced the construction of the Qutab Minar
in 1200 AD, but could only finish the basement.
▪ His successor, Iltutmush, added three more storeys.
▪ Firoz Shah Tughlak constructed the fifth and the last storey in 1368.

Q 89.D
• The monastic university of Nalanda is a mahavihara as it is a complex of several monasteries of various
sizes. Most of the information about Nalanda is based on the records of Xuan Zang—previously spelt as
‘Hsuan-Tsang’— which states that the foundation of a monastery was laid by Kumargupta I in the fifth
century CE. There is evidence that all three Buddhist doctrines— Theravada, Mahayana and
Vajrayana—were taught here and monks made their way to Nalanda and its neighbouring sites of Bodh
Gaya and Kurkihar from China, Tibet and Central Asia in the north, and Sri Lanka, Thailand, Burma and
various other countries from the south-eastern parts of Asia. Hence statement 1 is correct.
 Kanishka became a great patron of Mahayana Buddhism. He convened a council in Kashmir. The
members of the council composed 300,000 words which thoroughly explained the three pitakas or
collections of Buddhist literature. Kanishka got these entries engraved on sheets of red copper, enclosed
them in a stone receptacle and raised a stupa over it. Kanishka set up many other stupas to perpetuate the
memory of the Buddha. Hence statement 2 is correct.
 The progress of Mahayana Buddhism led to the composition of numerous avadanas. Most of these texts
were composed in what is known as the Buddhist-Hybrid Sanskrit. Their one objective was to preach
the teachings of Mahayana Buddhism to the people. Some of the important books of this genre were
the Mahavastu and Divyananda. Hence statement 3 is correct.

Q 90.A
• Kornish was a form of ceremonial salutation in which the courtier placed the palm of his right hand
against his forehead and bent his head. It suggested that the subject placed his head – the seat of the senses
and the mind – into the hand of humility, presenting it to the royal assembly.
 Chahar taslim is a mode of salutation which begins with placing the back of the right hand on the
ground, and raising it gently till the person stands erect, then he puts the palm of his hand upon the crown
of his head. It is done four (chahar) times. Taslim literally means submission.
• The forms of salutation to the ruler indicated the person’s status in the hierarchy: deeper prostration
represented higher status. The highest form of submission was sijda or complete prostration. Under Shah
Jahan these rituals were replaced with chahar taslim and zaminbos (kissing the ground).

Q 91.C
 Mahendravarman I
o He was a Pallava King who succeeded Simhavishnu and reigned for c. 600–630.
o He was a great patron of art and architecture and is known for introducing a new style to Dravidian
architecture, which is sometimes referred to as “Mahendra style.”
o He built temples at Panamalai, Mandagapattu and Kanchipuram.
o The inscription at Mandagapattu mentions Mahendravarman I with numerous titles such as:
▪ Vichitrachitta (curious-minded)
▪ Chitrakarapuli (tiger among artists)
▪ Chaityakari (temple builder)

Q 92.C
 Remnants of pre-historic rock paintings have been found on the walls of the caves situated in several
districts of Madhya Pradesh, Uttar Pradesh, Andhra Pradesh, Karnataka, Uttarakhand, J&K and
Bihar. These paintings have been found in North, South and East India.Hence statement 1 is not
correct.
• BHIMBETKA PAINTINGS: The caves of Bhimbetka were discovered in 1957–58 by eminent
archaeologist V.S. Wakankar. The themes of paintings found here are of great variety, ranging from
mundane events of daily life in those times to sacred and royal images. These include hunting,
dancing, music, horse and elephant riders, animal fighting, honey collection, decoration of bodies,
and other household scenes. Hence statement 2 is correct.
 The paintings at Bhimbetka can be divided into various parts based on style, technique and
superimposition:
o Upper Palaeolithic Period: The paintings of the Upper Palaeolithic phase were linear representations,
in green and dark red, of huge animal figures, such as bisons, elephants, tigers, rhinos and boars
besides stick-like human figures.
o Mesolithic Period: The largest number of paintings belong to this period. During this period the
themes multiply but the paintings are smaller in size. Hunting scenes predominate.
o Chalcolithic Period: The paintings of this period reveal the association, contact of the cave dwellers of
this area with settled agricultural communities of the Malwa plains.
 The artists of Bhimbetka used many colours, including various shades of white, yellow, orange, red
ochre, purple, brown, green and black. The paints were made by grinding various rocks and
minerals. They got red from haematite (known as geru in India). The green came from a green variety of
a stone called chalcedony. White might have been made out of limestone.Brushes were made of plant
fibre.
 LAKHUDIYAR PAINTINGS: found on banks of the River Suyal at Lakhudiyar,
UTTARAKHAND. Hence statement 3 is not correct.
o Lakhudiyar literally means one lakh caves. The paintings here can be divided into three categories:
man, animal and geometric patterns in white, black and red ochre. Humans are represented in stick-
like forms. A long-snouted animal, a fox and a multiple legged lizard are the main animal motifs.
Wavy lines, rectangle-filled geometric designs, and groups of dots can also be seen here. One of the
interesting scenes depicted here is of hand-linked dancing human figures. There is some
superimposition of paintings. The earliest are in black; over these are red ochre paintings and the last
group comprises white paintings.
 KASHMIR: From Kashmir two slabs with engravings have been reported.
• KARNATAKA AND ANDHRA PRADESH: The granite rocks of Karnataka and Andhra Pradesh
provided suitable canvases to the Neolithic man for his paintings. There are several such sites but more
famous among them are Kupgallu, Piklihal and Tekkalkota. Three types of paintings have been reported
from here—paintings in white, paintings in red ochre over a white background and paintings in red ochre.

Q 93.B
 Amil or munsif, a term which was used for a person who looked after collection of land revenue.
 Muqaddams were local village headmen.
 A number of villages known as Parganas were under the charge of Shiqdar who looked after the law
and order and general administration of the village.
 Barids were Intelligence Officers and Intelligence department was called as "Diwan-i-Barid".
 All these terms were common during medieval India, especially during the Mughal era.

Q 94.B
• Statement 1 is not correct: The early history of Harsha’s reign is reconstructed from a study
of Banabhatta, who was his court poet and who wrote a book called Harshacharita. Harisena was the
court poet of Emperor Samudragupta. He wrote a famous poem praising the bravery of Samudragupta that
is inscribed on the Allahabad Pillar.
 Statement 2 is correct: This can be supplemented by the account of the Chinese pilgrim Hsuan Tsang,
who visited India in the Seventh Century A.D. Hsuan-Tsang (sometimes transcribed Xuan Tsang or
Xuanzang) was a Chinese Buddhist monk who in 627 AD traveled overland from China to India to obtain
Buddhist scriptures. Hsuan-Tsang left detailed accounts of his travels, and also wrote about the interaction
between Chinese Buddhism and Indian Buddhism during the early Tang dynasty.
• Harshavardhana was one of the most important Indian emperors of the 7th Century. During the peak of his
reign, Harshavardhana’s empire extended from north India to the Narmada River in central India. His rule
was renowned for peace, stability, and prosperity, and attracted many artists and scholars from far and
wide.
 Ruling from 606 to 647 CE, Harshavardhana became the most successful emperor of the Pushyabhuti
dynasty until he was defeated by a South Indian ruler Pulakeshin II. The defeat of Harshavardhana
marked the end of the Pushyabhuti dynasty or Vardhana Dynasty.
• Harshavardhana’s kingdom was one of the earliest Indian kingdoms where we can see the practice of
feudalism. During the course of his rule, Harshavardhana built a strong army. Historical records suggest
that he had 100,000 strong cavalries, 50,000 infantry, and 60,000 elephants during the peak of his reign.
He was also a patron of literature and art.

Q 95.C
• Statement 1 is not correct: Akbar adopted Sher Shah’s administrative system. However, he did not find
it that much beneficial hence he had started his own administrative system. So in 1580, Akbar instituted a
new system called the dahsala, which formed part of his Land revenue administration.
 Statement 2 is not correct: Under this system, the average produce of different crops along with the
average prices prevailing over the last ten (dah) years was calculated.
 However, the state demand was stated in cash. One-third of the average produce was demand of state.
This was done by converting the state share into money on the basis of a schedule of average prices over
the past ten years.
 Akbar also introduced a new land measurement system (known as the zabti system) covering from
Lahore to Allahabad, including Malwa and Gujarat.
 Under the zabti system, the shown area was measured by means of the bamboos attached with iron
rings.
• The zabti system, originally, is associated with Raja Todar Mal (one of the nobles of Akbar), therefore,
sometimes, it is called as Todar Mal's bandobast. Todar Mal was a brilliant revenue officer of his time.
He first served on Sher Shah’s court, but later joined Akbar.
• Besides zabti system, a number of other systems of assessment were also introduced by Akbar. The most
common and, perhaps the oldest one was ‘batai’ or ‘ghalla-bakshi'.

Q 96.D
 All the pairs are correctly matched: Ur type of village was inhabited by peasant castes, it consisted of
the taxpaying residents of an ordinary village.
 The sabha type of villages consisted of brahmadeya villages or those granted to the brahmanas, and of
agrahara villages, The brahman owners enjoyed individual rights in the land but carried on their activities
collectively; and finally, the nagaram was found more commonly in trade centers, it consisted of villages
settled and dominated by combinations of traders and merchants.
 In some villages, the ur and the sabha were found together. Very large villages had two urs if this was
more convenient for their functioning. The working of these assemblies differed according to local
conditions.
 The ur was open to all male adults of the village but in effect the older members took a more prominent
part, some of them forming a small executive body for routine matters. The sabha had the same system
and in addition, had the power to constitute smaller committees of any size from among its members.

Q 97.A
 Pair 1 is not correctly matched: Chatu Vittalanatha was a Kannada poet who was patronized by Sri
Krishnadeva Raya (Vijaynagara empire). He is famous for his work Bhagavatha and Timmanna Kavi
who wrote his eulogy Krishnaraya Bharata was also patronized by Krishnadeva Raya.
 Pair 2 is correctly matched: Rajashekhara was an eminent Sanskrit poet, dramatist and critic. He lived
at the court of Mahipala (Pratiharas). Rajashekhara wrote the Kavyamimamsa. The work is essentially a
practical guide for poets that explains the elements and composition of a good poem. He is most noted for
the Karpuramanjari, a play written in Sauraseni Prakrit.
 Pair 3 is correctly matched: Svayambu was a great apabhramsha poet who lived at the court of
the Rashtrakutas.

Q 98.D
 Ashoka led a huge army against Kalinga, following the footsteps of his forefathers to expand his empire.
Kalinga war took place in 261 B.C. Kalinga offered stiff resistance to the Mauryan army. The whole of
Kalinga turned into a battle arena. However, the limited forces of Kalinga were no match for the
overwhelming Magadha army. Contrary to Ashoka's expectations, the people of Kalinga fought with such
great valour that on a number of occasions they came very close to a victory. The soldiers of Kalinga
perished in the battlefield fighting till their last breath for their independence. The victory ultimately
rested with Ashoka.
 The war took a tremendous toll on life and property. The 13th rock edict of Ashoka throws light on this
war. Ashoka appealed ideologically to the tribal people and the frontier kingdoms. The subjects of the
independent states in Kalinga were asked to obey the king as their father and to repose confidence in him.
The officials appointed by Ashoka were instructed to propagate this idea among all sections of his
subjects. The tribal peoples were similarly asked to follow the principles of dhamma (dharma). He no
longer treated foreign dominions as legitimate areas for military conquest. He took steps for the welfare
of men and animals in foreign lands, which was a new thing considering the conditions in those
times. Hence statement 1 is correct.
 Ashoka disapproved of rituals, especially those observed by women. He forbade killing certain birds and
animals, prohibited the slaughter of animals in the royal kitchen, and forbade the slaughter of animals in
sacrifices. Hence statement 2 is correct.
 Ashoka is important in history for his policy of peace, nonaggression, and cultural conquest. Although
Kautilya advised the king to be always intent on physical conquest, Ashoka followed quite the reverse
policy. He asked his successors to give up the policy of conquest and aggression, followed by the
Magadhan princes till the Kalinga war, and counselled them to adopt a policy of peace sorely needed
after a period of aggressive wars lasting for two centuries. He consistently adhered to his policy, for
though he possessed sufficient resources and maintained a huge army, he did not wage any war after the
conquest of Kalinga. Hence statement 3 is correct.

Q 99.C
• Buddhist literature describes the places associated with the Buddha’s life:
o where he was born (Lumbini)
o where he attained enlightenment (Bodh Gaya)
o where he gave his first sermon (Sarnath)
o where he attained nibbana (Kusinagara).
 Gradually, each of these places came to be regarded as sacred.
 So to mark the fact that Buddha had visited Lumbini, Asoka erected a pillar there about 200 years
after the time of the Buddha.
 The edicts are composed in non-standardized and archaic forms of Prakrit.
o Prakrit inscriptions were written in Brahmi and Kharosthi scripts, which even a commoner could read
and understand.
o The inscriptions found in the area of Pakistan are in the Kharosthi script.
o Other edicts are written in Greek or Aramaic.

Q 100.A
 The first to invade India were the Greeks, who are called the Indo-Greeks or Bactrian Greeks. In the
begining of the second century B.C., the Indo-Greeks occupied a large part of north-western India,
much larger than that conquered by Alexander. It is said that they pushed as far as Ayodhya and
Patliputra. But the Greeks failed to establish united rule in India. Two Greek dynasties ruled north-
western India on parallel lines at one and the same time.
 The most famous Indo-greek ruler was Menander (165-145 B.C.). He is also known by the name Milinda.
He had his capital at Sakala (Modern Sialkot) in Punjab; and he invaded the Ganga-Yammuna doab. He
was converted to Buddhism by Nagasena, who is also known as Nagarjuna. Menander asked Nagasena
many questions relating to Buddhism. These questions and Nagasena's answers were recorded in the form
of a book known as Milinda Panho or the Questions of Milinda.
 The Indo-Greek rule is important because of the large number of coins they issued. They were the first
rulers in India to issue coins which can be definitely attributed to the kings. This is not possible in
the case of the early punch-marked coins which cannot be assigned with certainty to any dynasty.
Hence option (a) is the correct answer.
 The Indo-Greeks were the first to issue gold coins in India, which increased in number under the Kushans.
 The Greek rule also introduced features of Hellinistic art in the north-west frontier of India. It
manifested itself in the form of an entirely new school known as Gandhara School of Art. Later it
spread over to Taxila, Mathura and Sarnath. This style of art chiefly concentrated on images and relics of
Buddha. They were made of either stone or stucco. Most of the ruins of this form of art found at Bimaran,
Dehri, Sakra and Hastanagar are now preserved in the museum of Peshawar in Pakistan.

You might also like